Archer Safety/Infection Control

Réussis tes devoirs et examens dès maintenant avec Quizwiz!

Which of the following is the most accurate education for injury prevention in the home of elderly clients? A. Use the handrail when going up and down the stairs, ensure robes or pants are held up if flowy, and wear comfortable slippers. B. Remove all throw rugs, remove furniture from all pathways, and wear comfortable/non-skid footwear. C. Use solid chairs without armrests, keep walkways clear, and use cordless phones. D. Install raised toilet seats, ensure that all sinks have throw rugs to prevent slipping on water, and use grab bars in the shower/bathroom.

Explanation Choice B is correct. Throw rugs, furniture in walkways, and slippery footwear are all fall risks for patients. Choice A is incorrect. Clients should be instructed to wear well-fitting shoes with non-skid soles. Choice C is incorrect. Solid chairs with armrests should be used. Choice D is incorrect. All throw rugs should be removed from the home. NCSBN Client Need Topic: Safe and Effective Care Environment, Subtopic: Safety and Infection Control; Preventing Falls Last Updated - 11, Nov 2021

Upon entering a client's room, the nurse finds the client lying on the floor. What is the first action the nurse should implement? A. Call for help to get the client back in bed B. Assist the client back to bed C. Establish if the client is responsive D. Ask the client what happened

Explanation Choice C is correct. Assessing if the patient is responsive is the primary concern of the nurse in this example. Choices A and B are incorrect. The client's responsiveness is a priority before moving the client. Choice D is incorrect. This answer choice would be the least important among the choices given. NCSBN Client Need Topic: Safe and Effective Care Environment, Subtopic: Safety and Infection Control - Falls Last Updated - 14, Feb 2022

The nurse is caring for a client who has generalized urticaria. The nurse should isolate the client using which of the following? A. Airborne precautions B. Droplet precautions C. Contact precautions D. Standard precautions Submit Answer

Explanation Choice D is correct. Generalized urticaria typically manifests when the client is experiencing an allergic reaction. This skin condition does not require isolation. The nurse should plan to care for this client using standard precautions. Choices A, B, and C are incorrect. These precautions are unnecessary in this patient with an allergic reaction. Last Updated - 13, Jan 2022

The nurse enters a client's room who is found on the ground. The nurse should perform which initial action? A. Assess the client's level of consciousness B. Examine the client for injuries C. Call the rapid response team (RRT) D. Palpate the client's carotid pulse Submit Answer

Explanation Choice A is correct. For a client found down on the ground, the nurse should immediately implement basic life support measures, including initially assessing the client's level of consciousness. If the client is unconscious, the nurse should stay with the client and shout for help. Choices B, C, and D are incorrect. Assessing the client's level of consciousness is the priority over assessing the client for injuries. Examining the client for injuries is a secondary assessment as their immediate concern is their overall stability. The RRT may need to be activated depending on the outcome of the client's level of consciousness. According to the BLS algorithm, the client's level of consciousness should be prioritized over the assessment of the carotid pulse.

What is the nurse's priority when a fire occurs in a client's room? A. Rescue the patient B. Extinguish the fire C. Sound the alarm D. Run for help Submit Answer

Explanation Choice A is correct. Patient safety is always the first priority. Choices B and C are incorrect. Sounding the alarm and extinguishing the fire are important things to do after the patient is safe. Choice D is incorrect. Calling for help, if possible, rather than running for help allows you to remain with the patient and is a more appropriate action. NCSBN Client Need Topic: Safe and Effective Care Environment, Subtopic: Safety and Infection Control - Fire Safety Last Updated - 15, Feb 2022

While at work on a medical-surgical floor, you hear an overhead announcement through the speaker system stating "Code Silver" three times, indicating an active shooter in the facility. Which of the following options is the most appropriate initial response for you to take? A. Close all the windows in the clients' rooms. B. Close all the clients' doors to their rooms. C. Perform a horizontal evacuation of your clients to avoid the gunman. D. Conduct a vertical evacuation of your clients to avoid the gunman.

Explanation Choice B is correct. Active shooter ("Code Silver" or, alternatively, "Code Gray" in some facilities) events are becoming increasingly common in healthcare facilities. Among the options listed, the most appropriate initial response would be to close all the client's doors to their rooms until you receive further instructions from the security officers or police officers. Closing the client's doors keeps the clients (and potentially any visitors) out of the gunman's view. Choice A is incorrect. The priority action must be closing the doors to the client's rooms to keep the client(s) and any visitor(s) out of the gunman's view. Closing the window(s) would consume valuable time and likely not make much difference, so the nurse should prioritize a more appropriate action. Choice C is incorrect. A horizontal evacuation is used to evacuate clients during a fire. More specifically, a horizontal evacuation refers to moving clients from one area to another unit or section on the same floor by safely passing through a fire door. A horizontal evacuation of the clients would not be performed in an active shooter situation, as the gunman's location would be an everchanging position. Choice D is incorrect. A vertical evacuation is used to evacuate clients during a fire. More specifically, a vertical evacuation refers to moving clients downward away from a threat on the upper floors by safely passing through a fire door (i.e., moving clients down a floor). A vertical evacuation of the clients would not be performed in an active shooter situation, as the gunman's location would be an everchanging position. Learning Objective During an active shooter event, prioritize shutting the door to the client's rooms. Additional Info An active shooting is typically over within 10 to 15 minutes, so advanced preparation on how to react is essential. According to the U.S. Department of Homeland Security, active shooters typically have no pattern or method for selecting their victims. Last Updated - 31, Aug 2022

The right brake on your client's wheelchair is not holding as strong as the left brake. What is your priority action? A. Ask the client if this just happened today. B. Immediately remove the wheelchair from use. C. Try to tighten the brake up with a simple tool. D. Call the physical therapist for another device.

Explanation Choice B is correct. Your priority action is to immediately remove the wheelchair from use as soon as you notice that the right brake on your client's wheelchair is not holding as strong as the left brake. Before any piece of medical equipment, including all assistive devices, is used the piece of equipment must be inspected; when there is any irregularity, as the right brake on your client's wheelchair not holding as strong as the left brake, it is not your role or responsibility to attempt to fix it because you are not competent to do so; therefore, you must immediately remove it from use and then notify the appropriate person or department and advise them that the piece of medical equipment needs a safety check and repair. Choice A is incorrect. Asking the client if this just happened today is irrelevant and unnecessary because it is not necessary to know when it occurred. Choice C is incorrect. You would not try to tighten the brake up with a simple tool because, not only is this not your role and responsibility, you are not equipped to do this. Choice D is incorrect. You would not call the physical therapist for another device as your priority action; however, after you have completed your priority action, you may call the physical therapist for a wheelchair. Last Updated - 03, Nov 2022

The charge nurse is reviewing room assignments and recognizes that only one private room is left. It would be appropriate to assign this room to the client with A. human immunodeficiency virus (HIV). B. delirium tremens who is agitated. C. disseminated herpes zoster. D. an implantable port that is accessed. Submit Answer

Explanation Choice C is correct. Disseminated herpes zoster requires airborne and contact precautions until lesions are dry and crusted. Choices A, B, and D are incorrect. HIV infection requires standard precautions. No isolation is necessary unless a superimposed infection is evident. Delirium tremens would require standard precautions. Finally, an implantable port client would not require isolation precautions. Additional Info Last Updated - 28, Oct 2022

The nurse observes sparks fly from a client's bathroom light. Which action should the nurse take first? A. Obtain a fire extinguisher B. Close the bathroom door C. Remove the client from the room D. Activate the fire alarm. Submit Answer

Explanation Choice C is correct. The initial action for the nurse to take is to remove the client from the room. This rescues the client from any danger. Choices A, B, and D are incorrect. Once the client has been removed from the scene, the nurse should activate the fire alarm, close the bathroom door, and obtain a fire extinguisher. Additional Info Last Updated - 10, Sep 2022

Which of the following situations is an example of negligence? A. The UAP (Unlicensed Assistive Personnel) fills a water basin with warm water while the patient with depression combs her hair. B. A nurse transcribes a new medication order: Questran powder 2 oz bid with wet food or one full glass of water. C. The nurse checks the distal pulses of a patient's legs two hours after they have returned from a cardiac catheterization. D. The nurse observes a UAP enter the room of a patient on contact precautions wearing gloves and a gown. Submit Answer

Explanation Choice C is correct. The nurse should have checked the patient's distal pulse immediately after the cardiac catheterization. Negligence and professional negligence are examples of unintentional torts that may occur in the health care setting. Negligence is misconduct or practice that is below the standard expected of an ordinary, reasonable, and prudent person. Such conduct places another person at risk for harm. Both nonmedical and professional individuals can be liable for negligent acts. Gross negligence involves an extreme lack of knowledge, skill, or decision-making that the person clearly should have known would put others at risk for harm. Malpractice is "professional negligence," that is, negligence that occurred while the person was performing as a professional. Malpractice applies to primary care providers, dentists, lawyers, and generally includes nurses. In some states, nurses cannot be sued for malpractice, only professional negligence. The terms malpractice and professional negligence are often used interchangeably. Six elements must be present for a case of nursing professional negligence to be proven: The nurse must have (or should have had) a relationship with the client that involves providing care and following an acceptable standard of care. Such duty, for example, is evident when the nurse has been assigned to care for a client in the home or hospital. A nurse also has a general duty of care, even if not explicitly assigned to a client, if the client needs help. There must be a standard of care that is expected in the specific situation but that the nurse did not observe. For example, something was done that should not have been done, or nothing was done when it should have been done. This is the failure to act as a reasonable, prudent nurse under the circumstances. The standard can come from documents published by national or professional organizations, boards of nursing, institutional policies and procedures, or textbooks or journals, or expert witnesses may state it. A link must exist between the nurse's act and the injury suffered. It must be proven that the harm occurred as a direct result of the nurse's failure to follow the standard of care and that the nurse could have (or should have) known that failure to comply with the standard of care could result in such harm. Harm or Injury. The client or plaintiff must demonstrate some type of damage or injury (physical, financial, or emotional) as a result of the breach of duty owed to the client. The plaintiff will be asked to document the bodily injury, medical costs, loss of wages, pain/suffering, and any other damages. Damages. If professional negligence caused the injury, the nurse is held liable for damages that may be compensated. The goal of awarding damages is to assist the injured party to his or her original position as far as financially possible. Choices A, B, and D are incorrect. None of these answer choices are examples of negligence on behalf of the nurse. NCSBN Client Need Topic: Safe and Effective Care Environment, Subtopic: Coordinated Care, Areas of Liability Last Updated - 15, Feb 2022

You are taking care of a 79-year-old African American woman on the general medical-surgical floor. While performing your assessment, you notice that she has a very flat affect, inferior communication skills, and seems to only be capable of concrete thinking. She even uses words you have never heard before. Which of the following illnesses do you suspect this woman may suffer from? A. Bipolar disorder B. Paranoid personality disorder C. Schizophrenia D. Panic disorder Submit Answer

Explanation Choice C is correct. These signs and symptoms are highly suggestive of schizophrenia. The nurse should speak with the healthcare team about her concerns and request a psychiatric consultation if not already done. Signs and symptoms of schizophrenia include: an inappropriate, flat, or blunted affect, focus on their inward world instead of reality, looseness of associations, echolalia, neologisms, word salad, delusions, hallucinations, and the inability to use abstract thinking skills. Choice A is incorrect. These signs and symptoms do not suggest bipolar disorder. Bipolar disorder manifests when a patient alternates between mania and depression. Symptoms of mania include labile emotions, delusions, insomnia, poor judgment, and constant activity. Symptoms of depression include anhedonia, weight loss or gain, sleep disturbances, and negative thoughts. Choice B is incorrect. These signs and symptoms do not suggest paranoid personality disorder. Signs and symptoms of paranoid personality disorder include anger and rage when appropriate, hypersensitive, and unable to relax, always jealous for no reasonable reason, paranoid thoughts/beliefs, as well as being suspicious and distrusting of all others with no purpose. Choice D is incorrect. These signs and symptoms do not suggest panic disorder. This is a relatively common disorder but often goes untreated. Signs and symptoms usually start to manifest when the patient is in their mid to late 20s. It starts with less severe and less frequent panic attacks but escalates to debilitating attacks that can occur weekly. The patient can even think they have a heart attack due to the severity of the symptoms such as chest pain, shortness of breath, and unrelenting anxiety. NCSBN Client Need: Topic: Psychosocial Integrity; Subtopic: Psychiatric Nursing Last Updated - 25, Jan 2022

Upon entering a patient's room, the nurse finds the patient lying on the floor of the bathroom. When documenting the incident on the patient's medical record, the nurse should include all of the following information, except? A. Provide an objective description of what happened. B. Report what the nurse observed. C. Note that an occurrence report was completed. D. Describe follow-up actions taken.

Explanation Choice C is correct. When documenting an incident, you do not mention the occurrence report in the patient's medical record. In the chart, you would provide an objective description of what happened citing your observations, and then describe the follow-up actions taken. Choices A, B, and D are incorrect. These are components of the incident report that should be included. Therefore, these are incorrect answers to the question being asked. Last Updated - 10, Feb 2022

Which of the following must NOT be included in an incident or accident form or report? A. The name of the person completing the report. B. The name of the client and if anyone was injured. C. The location of the incident or accident. D. An explanation of what may have led to the incident. Submit Answer

Explanation Choice D is correct. An explanation of what may have led to the incident is NOT included in an incident or accident form or report. Comments about what may have led to the event are speculative and not factual. Only facts, such as the condition of the floor after a client fall, are documented on an incident or accident form or report. Choice A is incorrect. The name of the person completing the report must be included in an incident or accident form or report. Choice B is incorrect. The name of the client and anyone that was injured must be included in an incident or accident form or report. Choice C is incorrect. The location of the incident or accident must be included in an incident or accident form or report. Last Updated - 11, Feb 2022

The nurse is planning a staff development conference about restraints. Which of the following information should the nurse include? Select all that apply. A. Mittens are not restraints if untethered and the client is physically able to remove the mitt. B. Elbow restraints may allow a client to remove abdominal or urinary medical devices. C. Soft wrist restraints should be removed one at a time if a client is violent. D. Belt restraints may be prescribed on an as-needed basis. E. Belt restraints should be applied over a client's clothing garments.

Explanation Choices A, B, C, and E are correct These statements are true and should be included in the conference. Mittens are not considered restraints if they are untethered, and the client may be able to remove the mitt. Elbow restraints make it difficult to remove a medical device near the face or neck. It does not impede the removal of abdominal or urinary medical devices. Belt restraints should be applied over a client's clothing and secured to the bedframe. Significant impairment to a client's skin may result if the belt is directly applied over the skin. Choice D is incorrect. Physical restraints, no matter the type, cannot be utilized on an as needed (PRN) basis. Physical restraint usage should be appropriately justified based on the client's behavior at that time of the demonstration. This information should not be included because it is incorrect. Additional Info Restraints should be used as a last resort if alternative methods are not effective. A nurse should never threaten a client with restraints. This is considered assault. The nurse may place a client who is violent in restraints without an order from the primary healthcare provider (PHCP). If this was to occur, the nurse has one hour to inform the provider and obtain an order. Restraints are never as needed (PRN). They should be discontinued at the earliest possible time. When restraining a client, the reason for the restraint must be explained to the client and the behavior the client needs to demonstrate for the restraints to be discontinued. The nurse should observe the client at frequent intervals to offer nutrition & toileting, assess their behavioral status, obtain vital signs, and provide range of motion. These intervals are determined by the facility and the type of restraint—the more restrictive the restraint and the younger the client, the more frequent assessment. Restraints must be able to quickly be removed via a quick-release buckle (knots are no longer recommended). The nurses' documentation must be comprehensive, describing the reasoning for the restraints, alternatives utilized, the education provided to the client, the type of restraint utilized, how it was secured, and the ongoing behavior necessary to continue the restraint. The nurse should also document the intervals at which the restraints were released. Last Updated - 13, Jun 2022

The nurse is caring for a patient with dementia who exhibits increased confusion during the evenings and frequently attempts to get out of bed. Which interventions would be appropriate for the nurse to implement before resorting to physical restraints? Select all that apply. A. Initiate toileting schedule B. Place patient near the nurses' station C. Keep one bedrail fully up and the other side half up D. Implement electronic bed alarm

Explanation Choices A, B, and D are correct. A: Patients with dementia often experience increased confusion during evenings (sundowning effect) and may not be able to effectively communicate their needs. Cognitively impaired patients who frequently try to get out of bed at night may be attempting to get to the bathroom. This nurse and other members of the patient's care team should implement a toileting schedule for this patient to reduce unsafe attempts and prevent incontinence. B: Placing this patient near the nurses' station would be appropriate since it would allow for faster assistance/alarm response and more frequent assessments. D: Electronic bed and chair alarms would be an appropriate, non-restraint intervention for this patient that would help to reduce the risk of falls by signaling to staff that the patient may be attempting to get out of bed. Choice C is incorrect. Full-length bedrails are a form of physical restraint when used to prevent the patient from getting out of bed. Confused patients may not recognize the bedrail as a reminder to stay in bed, and studies have shown that routine use of these barriers can increase the risk of falls and injury. Last Updated - 15, Feb 2022

You work in a community clinic in a large city. There has been a recent outbreak of meningococcal meningitis at the local university and students who have been in contact with the sick students have been advised by public health officials to obtain prophylactic treatment. Which of the following would be helpful in preventing this disease? Select all that apply. A. Amoxicillin B. Ciprofloxacin C. Rifampin D. Meningococcal conjugate vaccine Submit Answer

Explanation Choices B, C, and D are correct. Meningococcal meningitis is transmitted through respiratory droplets from infected individuals. After exposure, symptoms will usually appear within 3 to 4 days. The CDC does not recommend universal prophylaxis during an outbreak, but prophylactic treatment should be provided for individuals in close contact with the infected patients. A single dose of ciprofloxacin or four doses of rifampin over two days can be useful in preventing the acquisition of the disease. Meningococcal conjugate vaccine (MCV4) is the preferred vaccine for at-risk individuals in this group. College students often receive this vaccination before attending school. Choice A is incorrect. Amoxicillin is not a treatment that will provide chemoprophylaxis. NCSBN Client Need Topic: Pharmacological and Parenteral Therapies, Sub-Topic: Expected Actions/Outcomes, Neurologic Last Updated - 22, Nov 2021

Documenting the statement, "Normal speech is audible", would be a normal finding for which speech quality? A. Loudness B. Articulation C. Fluency D. Quality Submit Answer

Explanation Choice A is correct. The term "audible" refers to the loudness of something. Characteristics of speech to evaluate include rate, rhythm, loudness, fluency, quantity, articulation, content, and pattern. Choice B is incorrect. Articulation refers to the production and use of sounds. Choice C is incorrect. Fluency is a speech-language pathology term that means the smoothness or flow with which sounds, syllables, words, and phrases are joined together when babbling. Choice D is incorrect. Speech quality refers to the characteristic features of an individual's voice. NCSBN Client Need Topic: Health Promotion and Maintenance, Subtopic: Health Assessment Last Updated - 23, Jan 2022

A patient is rushed to the emergency department after being exposed to radioactive materials in a workplace accident. What should be the initial action of the nurse? A. Strip the patient of all clothing and decontaminate him. B. Ask the patient what happened during the accident. C. Decontaminate the room where the patient was staying. D. Save the clothing for analysis. Submit Answer

Explanation Choice A is correct. Removing any radioactive material from the patient should be the first priority of the nurse. Choice B is incorrect. The nurse should investigate what happened after all contaminants have been removed from the patient. Choice C is incorrect. The primary responsibility of the nurse is the patient, not the room. The room can be decontaminated once the patient has been stabilized and treated. Choice D is incorrect. The nurse can save the clothing for analysis, but only after the client has been stabilized. Last Updated - 07, Feb 2022

When orienting an older patient to the safety measures in his hospital room. What is the priority component of this admission routine? A. Explain how to use the telephone B. Introduce the patient to his roommate C. Review the hospital policy on visiting hours D. Explain how to operate the call light Submit Answer

Explanation Choice D is correct. Knowing how to use the call light is a safety priority. Choices A, B, and C are incorrect. Knowledge of how to use the telephone, meeting a roommate, and knowing the hospital policy about visiting hours will not necessarily prevent accidental injury. NCSBN Client Need Topic: Safe and Effective Care Environment, Subtopic: Safety and Infection Control - Orienting the Patient to Surroundings Last Updated - 11, Nov 2021

The nurse has been assigned to provide care for a group of patients that includes a patient with Mycoplasma pneumonia and a patient with Clostridium difficile diarrhea. What approach should the nurse use to best protect against the transmission of these infections to other patients? A. Perform hand hygiene before, after, and between providing direct patient care. B. Wear examination gloves whenever in direct contact with any patient. C. Cleanse equipment such as thermometers or stethoscopes between patients. D. Maintain a distance of 3 feet away from patients who are coughing. Submit Answer

Explanation Choice A is correct. The Centers for Disease Control and Prevention cite handwashing as the single most effective way to prevent the transmission of disease. The effectiveness of other measures is dependent upon the foundation of appropriate hand hygiene. Choice B is incorrect. While many nurses do use exam gloves whenever in contact with patients, this will not be effective without practicing hand hygiene. Choice C is incorrect. While cleaning shared equipment will help prevent transmission of disease, it is not the most effective measure and is ineffective if the nurse had not practiced hand hygiene. Choice D is incorrect. Maintaining a distance of three feet from others who are coughing is recommended; however, this is not always feasible when providing patient care. Again, hand hygiene is the most effective preventative measure. Last Updated - 24, Jan 2022

The charge nurse is making room assignments for four new patients being admitted to the unit. There is one private room available. The patient who should be assigned to the private room is the patient with: A. Seizures B. Diabetes C. Hyperthyroidism D. Clostridium difficile Submit Answer

Explanation Choice D is correct. The patient with Clostridium difficile (C. diff) should be placed in a private room. This infection is a significant health threat that can lead to death in sick or immunocompromised patients. Clostridium difficile symptoms include persistent watery diarrhea, abdominal pain, nausea, fever, and appetite loss. It is spread quickly from one person to another, so exceptional hand hygiene must be practiced by anyone coming into contact with the patient. Also, personnel must use gloves and gowns when caring for these patients. Full contact-enteric precautions must be practiced. Contact enteric precautions include all contact precautions plus washing hands with soap and water when leaving the patient room. Choices A, B, and C are incorrect. Patients with seizures, diabetes, and hyperthyroidism do not require placement in a private room since they do not pose a threat to others. NCSBN Client Need Topic: Safety and Infection Control; Management of Care; Sub-Topic: Standard Precautions/Transmission-Based Precautions Last Updated - 27, May 2021

The nurse evaluates a student's ability to appropriately apply personal protective equipment (PPE). It would indicate effective teaching if the student dons PPE in which order? Place the steps in the appropriate order. Mask Gown Goggles Gloves Submit Answer

Explanation The sequence for donning (applying PPE) is gown, mask (respirator), goggles, and gloves. Additional Info ✓ The nurse can prevent disease transmission by exercising meticulous hand hygiene, educating others, and correcting inappropriate actions. ✓ The appropriate donning sequence of PPE is the gown, mask or respirator, goggles or face shield, and gloves. ✓ The appropriate sequence for doffing PPE is gloves, face shield or goggles, gown, and mask or respirator. Last Updated - 21, Jan 2023

The nurse is reviewing client room assignments. Which room assignment requires modification? See the image below. A. Room 1 B. Room 2 C. Room 3 D. Room 4

Explanation Choice A is correct. Lyme disease is not transmitted human-to-human (a tick transmits it). This client should be placed on standard precautions as contact precautions are inappropriate. This client's room assignment requires follow-up. Choices B, C, and D are incorrect. Varicella requires a client to be placed on contact and airborne precautions until the lesions are dry and crusted. Mumps requires droplet precautions. HIV does not require isolation precautions. Additional Info Last Updated - 09, Nov 2022

You are a nurse working on an adult medical-surgical floor when you hear "Code Pink" repeated three times over the hospital-wide speaker system, indicating an infant or child abduction. Since you know the hospital has infant/child abduction drills every two months, and you are working in an area without infants or pediatric clients, you should do which of the following? A. You must respond and perform your role in this "Code Pink." B. Ask the unit secretary to respond to the "Code Pink" for you. C. Ignore the "Code Pink" because you are caring for clients. D. Ignore the "Code Pink," as you are not staffing in obstetrics, the nursery, NICU, or pediatrics.

Explanation Choice A is correct. Each and every hospital employee on duty during a "Code Pink" would immediately respond and perform their specified role per hospital policy, regardless of the unit the individual was staffing. Here, the fact that you are staffing the adult medical-surgical area has no bearing on whether you have an obligation to respond. You must always respond to each Code as if it were an actual emergency. Choice B is incorrect. Asking the unit secretary to respond to the "Code Pink" for you is inappropriate, as you have a duty to respond to this Code per hospital policy. Additionally, the unit secretary more than likely also has their own duty to respond to the "Code Pink" under the hospital policy as well. Choice C is incorrect. You have a duty to respond to the "Code Pink" and would therefore not ignore the Code. While it is understood you are caring for clients, you have a role and responsibility during the "Code Pink." Choice D is incorrect. Staffing in a client care area other than obstetrics, the nursery, NICU, or pediatrics is not an excuse to disregard a "Code Pink" or not respond per hospital policy. During a "Code Pink," all hospital employees are required to respond per hospital policy. Learning Objective Recognize that a "Code Pink" announcement indicates an infant/child abduction within the facility and requires the response of all hospital employees per hospital policy. Additional Info You cannot and should not ever assume any "Code Pink" (or any other code) is a practice drill. Although each facility has individual "Code Pink" policies and procedures, most facilities implement the following during a "Code Pink" response: All external doors are locked upon activation of a Code Pink, with no one allowed to leave the facility until the resolution of the Code. Employees must proceed to their designated location or begin immediately searching the floor per their department response guidelines. Staff are to pay close attention to exits/stairwells and quick exiting areas and to observe for anyone carrying a child, bags, oversized coats, or containers large enough to hold or conceal an infant or child. Notify Security immediately of any person with suspicious or unusual behavior.

The nurse has just given an intradermal injection of PPD to a client in the clinic when she accidentally sticks herself in the finger with the used needle. What is the initial action of the nurse? A. Fill out an incidence occurrence report. B. Wash the area with soap and water right away. C. Ask the client if he has HIV or hepatitis. D. Put an antibiotic cream and bandage over the site. Submit Answer

Explanation Choice B is correct. The initial action of the nurse should be to wash the area with soap and water first then try to squeeze the area to make it bleed. Choice A is incorrect. The incident should be documented entirely; however, the nurse should care for the wound first. Choice C is incorrect. The nurse should not directly ask the client. The nurse may refer to the client's chart or ask the client to have his blood drawn for testing. Choice D is incorrect. The first puncture site would not need an antibiotic ointment. Additional Info Last Updated - 17, Oct 2021

The nurse is discussing infection control with a group of nursing students. It would be correct to state that the contact precautions with alcohol-based hand hygiene measures should be sufficient for which of the following conditions? Select all that apply. A. Respiratory Syncytial Virus (RSV) B. Mumps C. Rubella D. Varicella E. Scabies F. Clostridium difficile Submit Answer

Explanation Choices A and E are correct. Conditions requiring the usual contact precautions include Respiratory Syncytial Virus (RSV) and Scabies. RSV is spread through contact with surfaces and contact with infectious droplets. Droplet precautions are not necessary for RSV. The CDC recommends standard and contact precautions for RSV. Other conditions requiring the usual contact precautions include: Mucocutaneous Herpes Simplex Virus (HSV) Methicillin-Resistant Staphylococcus Aureus (MRSA) Pediculosis Contact precautions protect against organisms that spread through contact with the patient or the patient's environment. Personal Protective Equipment (PPE) required for contact precautions include gloves and a gown. In contact precautions, the nurse/visitor must perform hand hygiene before entering the patient's room and after leaving the room. One can clean hands either with an alcohol-based hand sanitizer or soap and water. However, in diarrheal illnesses such as Clostridium difficile or norovirus, one must follow contact precautions but with an additional requirement. It requires visitors to perform hand hygiene by cleaning hands with soap and water after leaving the patient room. These special contact precautions are referred to as contact enteric precautions, and the isolation sign on the patient room must clearly state this requirement. Alcohol-based disinfectants do not kill Clostridium spores or norovirus. Therefore, soap and water are mandatory to clean hands upon leaving the patient's room. Choices B and C are incorrect. Mumps and Rubella require droplet precautions. Rubella (German measles) and Rubeola (Measles) sound similar, but they are two different diseases. Rubeola (Measles) spreads by airborne route and needs airborne isolation, whereas Rubella needs droplet isolation. Choice D is incorrect. Varicella-Zoster Virus (VZV) causes Chickenpox and Shingles. Varicella (Chickenpox) needs both contact and airborne isolation precautions, not just contact precautions. Both contact and airborne precautions are used in Varicella and "disseminated" Herpes Zoster (shingles) until all the lesions dry and crust over. In contrast, standard precautions alone are sufficient for "localized" Herpes Zoster in immunocompetent patients if the lesions can be contained/covered. Choice F is incorrect. Clostridium difficile is a bacteria that causes diarrhea and is highly contagious. It spreads by contact and fecal-oral route. Contact precautions are certainly used to prevent the spread of Clostridium difficile. However, an additional requirement of cleaning hands with soap and water upon leaving the patient's room is mandatory. To standardize infection control practices and to specify this additional hand hygiene requirement, many hospitals have adopted special signage, "contact enteric precautions".

While working on a pediatric floor, you have a patient diagnosed with pertussis. Which image below indicates the correct precautions the nurse needs to place the patient on? A .

Explanation Choice C is correct. Droplet precautions are necessary for the patient with pertussis. This means that the client needs a private room, or a room shared only with another client with pertussis. Staff should and visitors must wear a mask when entering the room. A mask must be placed on the client if they need to leave the room. Choices A, B, and D are incorrect. These types of precautions are not applicable to patients with pertussis. NCSBN Client Need Topic: Health Promotion and Maintenance Subtopic: Health Promotion/Disease Prevention Last Updated - 12, Feb 2022

The nursing diagnosis "[a]t risk for insufficient vascular perfusion" would most apply to which of the following clients? A. An adolescent client undergoing an expected maturational growth spurt B. A 6-year-old pediatric client with a leg recently placed in a cast following a greenstick fracture C. A 76-year-old female client with urinary and fecal incontinence D. A 42-year-old male client who recently sprained his ankle while playing basketball and wrapped the affected ankle in an elastic bandage Submit Answer

Explanation Choice B is correct. The nursing diagnosis "[a]t risk for an alteration in vascular perfusion" is most applicable to this client, as the recently casted extremity following the greenstick fracture places this client at risk for decreased tissue and vascular perfusion, primarily compartment syndrome. Compartment syndrome is always a concern for a client with a newly placed cast. In compartment syndrome, tissue pressure (often due to swelling) occurs within a confined space (i.e., within the cast), leading to restricted blood flow and eventually ischemia. Compartment syndrome is always considered a medical emergency and requires prompt medical intervention. Failure to intervene quickly can result in ischemia, possibly leading to irreversible damage to the tissue(s). Choice A is incorrect. An anticipated maturational growth spurt does not place an adolescent client at risk for insufficient vascular perfusion. Choice C is incorrect. A 76-year-old female client who is incontinent of both urine and feces is not at risk for insufficient vascular perfusion based on the incontinence alone. Although other age-related issues may put this client at risk for insufficient vascular perfusion, nothing about the incontinence places the client at increased risk. Of note, the client is likely at risk for other potential complications, such as skin integrity issues. Choice D is incorrect. Although the 42-year-old male client who recently sprained his ankle while playing basketball and wrapped the affected joint in an elastic bandage is indeed "[a]t risk for an alteration in vascular perfusion," the question asks for the client who is at the most risk for an alteration in vascular perfusion. Although an elastic wrap could alter circulation or perfusion to the tissues, the elastic contained in the wrap is more likely to expand in the event of swelling as opposed to a restrictive cast as worn in the client referenced in Choice B. Learning Objective Identify which client is most at risk for insufficient vascular perfusion. Additional Info A greenstick fracture occurs when a bone is angulated beyond the bending limits, resulting in the compressed side bending and the tension side failing, causing an incomplete fracture similar to the break observed when a green stick breaks. The pliable bones of the growing child are more porous than the adult's, allowing them to bend, buckle, and break in a "greenstick" manner. Compartment syndrome is assessed utilizing the five P's: pain, paralysis, paresthesia, pallor, and pulselessness. Time is of the essence when dealing with compartment syndrome or any other type of insufficient vascular perfusion. Elevating a limb suspected of developing or having developed compartment syndrome is contraindicated. Doing so would reduce the already impaired blood flow by decreasing arterial flow. Last Updated - 01, Aug 2022

A patient is scheduled to undergo a CT scan with intravenous contrast dye. Which of the following actions, if performed by the new nurse, would require follow-up by the supervising nurse? A. Encourage fluids when the patient returns from the scan. B. Confirm that the consent form is signed. C. Raise the side rails of the patient's stretcher during transport. D. Cancel the CT scan if the patient reports a shellfish allergy. Submit Answer

Explanation Choice D is correct. This action will require follow-up. It is inappropriate to cancel the CT scan if the patient has a history of shellfish allergy. Previously, it was felt that allergy to shellfish/seafood (because they contain iodine) and allergy to topical iodinated products conferred cross-allergy with iodine-containing contrast dyes. Iodine is found ubiquitously in the form of thyroid hormones, and there is no such thing as an allergy to systemic iodine. Minor skin reactions to topical iodine do not confer cross-allergy to IV contrast dyes. There is no evidence to support this notion; therefore, current guidelines do not suggest treatment plan modification based on a history of shellfish or seafood allergy alone. According to the American College of Radiology (ACR) Manual on contrast media, there is no evidence to support the continuation of this old practice of inquiring specifically into a patient's history of "allergy" to seafood, especially shellfish. Choices A, B, and C are incorrect. These actions are most appropriate to ensure patient safety and do not require follow-up. Contrast dye can be toxic to the kidneys and may result in renal failure. The risk of contrast-induced nephropathy is higher in those who are dehydrated, those on diuretics, and those with underlying co-morbidities such as diabetes. The nurse needs to encourage adequate fluid intake (choice A) to promote the flush out of the contrast dye. The consent form must always be signed (choice B) before administering IV contrast to ensure safety and accuracy. During pre-screening, the consent form has a section that inquires about the patient's history of allergies, including any prior reactions to x-ray contrast. Safety measures must be ensured during patient transportation (e.g., transport to the x-ray department). During patient transport, specific safety measures include locking the equipment in a stopped position and keeping side rails up on stretchers (choice C). Learning Objective Recognize that shellfish allergy is no longer a contraindication to administering the IV contrast dye. Additional Info Before IV contrast administration, prescreening must include questioning the patient regarding prior reactions to contrast dye or allergies to medications/substances. Any allergy (not specifically shellfish) may increase the risk of having an IV contrast dye reaction. If there is a history of severe allergies or prior reaction to contrast dye, such patients may be premedicated with diphenhydramine and steroids. Shellfish allergy is not a contraindication. Students who would like to learn more about this should refer to this article at https://www.ncbi.nlm.nih.gov/pmc/articles/PMC3964187/

The nurse is applying soft wrist restraints to a client who is violent towards the nursing staff. Which actions by the nurse are appropriate? Select all that apply. A. Places a pair of scissors at the bedside for emergent discontinuation. B. Positions the client supine after applying both wrist restraints. C. Releases both restraints at the same time, every two hours. D. Informs the client of the behavior necessary to demonstrate to end the restraints. E. Ensures two fingers can be placed under each restraint. Submit Answer

Explanation Choices D and E are correct. The nurse should always inform the client of the reason for being placed in the restraint and what behavior needs to be demonstrated to terminate the restraints. It is appropriate for the nurse to ensure two fingers can be placed under each restraint as this verifies that it will not cause damage to the client's skin. Choices A, B, and C are incorrect. These actions are inappropriate. Scissors should not be placed at the bedside for a client who is violent, and they are also unnecessary because restraints should be disengaged with a quick-release buckle. Placing the client completely supine is not recommended. This position makes the client feel vulnerable and may cause humiliation. Semi-Fowlers' to high-Fowlers are recommended positions. A client should never be placed prone or supine while restrained. For a client who is violent, the nurse should remove one restraint at a time and/or have staff assistance while removing restraints. Additional Info Restraints should be used as a last resort if alternative methods are not effective. A nurse should never threaten a client with restraints. This is considered assault. The nurse may place a client who is violent in restraints without an order from the primary healthcare provider (PHCP). If this was to occur, the nurse has one hour to inform the provider and obtain an order. Restraints are never as needed (PRN). They should be discontinued at the earliest possible time. When restraining a client, the reason for the restraint must be explained to the client and the behavior the client needs to demonstrate for the restraints to be discontinued. The nurse should observe the client at frequent intervals to offer nutrition & toileting, assess their behavioral status, obtain vital signs, and provide range of motion. These intervals are determined by the facility and the type of restraint—the more restrictive the restraint and the younger the client, the more frequent assessment. Restraints must be able to quickly be removed via a quick release buckle (knots are no longer recommended). The nurses' documentation must be comprehensive, describing the reasoning for the restraints, alternatives utilized, the education provided to the client, the type of restraint utilized, how it was secured, and the ongoing behavior necessary to continue the restraint. The nurse should also document the intervals at which the restraints were released. Last Updated - 18, Dec 2022

One of the complications associated with the improper use of crutches is: A. Axillary nerve damage B. Solar plexus nerve damage C. Carpal tunnel syndrome D. Trigeminal nerve damage Submit Answer

Explanation Choice A is correct. One of the complications associated with the improper use of crutches is axillary nerve damage. This damage occurs when the user of the crutches rests and supports their natural weight with their underarms rather than on their hands, and the hand rests on the crutches. Choice B is incorrect. Solar plexus nerve damage is not one of the complications associated with the improper use of crutches. The solar plexus, which is the most massive bundle of nerves in the human body and is also referred to as the celiac plexus. It is located in the abdominal area and is affected/damaged by a blow to the abdomen, not associated with the improper use of crutches. Choice C is incorrect. Carpal tunnel syndrome, also referred to as repetitive stress syndrome, results from the repetitive action of the hands and wrists and not from the improper use of crutches. Choice D is incorrect. Trigeminal nerve damage is not one of the complications associated with the improper use of crutches. Trigeminal nerve damage can occur as the result of a physiologically pathological cause like a tumor, multiple sclerosis, and causes other than the improper use of crutches.

The nurse is admitting a client diagnosed with hepatitis B. The nurse would be able to cohort the client in the same room with which of the following clients? A client with A. heart failure receiving diuretics B. bacterial meningitis receiving antibiotics C. prostate cancer receiving brachytherapy D. varicella prescribed antivirals Submit Answer

Explanation Choice A is correct. Although hepatitis B needs to be reported to the public health department, a client with hepatitis B does not need to be isolated. An appropriate client to room with would be an individual receiving intravenous diuretics for heart failure as this client does not have any transmissible pathogens. Choices B, C, and D are incorrect. A client with bacterial meningitis requires droplet precautions, a client receiving brachytherapy requires a private room, and airborne isolation must be initiated for an individual with varicella. Additional Info A nurse may cohort a client if the pathogen they have is the same. A patient requiring airborne isolation must have a private room with their door kept closed while maintaining negative pressure. Cohorting a client with an airborne pathogen is strongly discouraged. Last Updated - 23, Jun 2022

While performing in-room charting on a patient. The nurse smells smoke from the bathroom upon entering the lavatory. She finds a trashcan on fire. After evacuating the patients from the room. What is her priority action? A. Activate the fire alarm. B. Extinguish the fire. C. Contact the nursing supervisor. D. Close the door to the bathroom. Submit Answer

Explanation Choice A is correct. Upon finding a fire, the nurse's first step should be to evacuate any patients who are in immediate danger. Once this step is completed, she must activate the fire alarm. Choice B is incorrect. The nurse doesn't need to focus on extinguishing the fire until the fire alarm has been activated and the fire confined. Choice C is incorrect. Contacting the nursing supervisor is not a priority during a fire-related emergency. Choice D is incorrect. Confining the fire is not necessary until the fire alarm has been activated. NCSBN client need Topic: Safety and Infection Control, Emergency Response Plan Last Updated - 06, Nov 2021

The nurse is educating the parents of a child who plans on riding their bicycle. Which statements, if made by the parents, indicate effective understanding? A. "I should tell my child should ride against the traffic pattern." B. "I should instruct my child to walk their bike through busy intersections." C. "Wearing a helmet is only necessary when my child is riding near a busy intersection." D. "My child can ride their bike barefoot as long as it's short distances." Submit Answer

Explanation Choice B is correct. A child should walk their bike through busy intersections to reduce their risk of being hit by an automobile. Choices A, C, and D are incorrect. These statements require follow-up because they do not indicate effective teaching. When riding a bicycle, it should be directed with the traffic flow. Any individual riding a bicycle should always wear a helmet, no matter the distance they plan on riding. To avoid serious foot injuries, the child should always wear proper fitting shoes while bicycling. Additional Info • Always wear a properly fitted helmet that the US Consumer Product Safety Commission approves; replace a damaged helmet. • Ride bicycles with traffic and away from parked cars. • Ride single file. • Walk bicycles through busy intersections and at crosswalks. • Give hand signals well in advance of turning or stopping. • Keep as close to the curb as practical. • Watch for drain grates, potholes, soft shoulders, and loose dirt or gravel. • Keep both hands on handlebars except when signaling. • Never ride double on a bicycle. • Do not carry packages that interfere with vision or control; do not drag objects behind bike. • Watch for and yield to pedestrians. • Watch for cars backing up or pulling out of driveways; be especially careful at intersections. • Look left, right, then left before turning into traffic or roadway. • Never hitch a ride on a truck or other vehicle. • Learn the rules of the road and respect for traffic officers. • Obey all local ordinances. • Wear shoes that fit securely while riding. • Wear light colors at night, and attach fluorescent material to clothing and bicycle. • Be certain the bicycle is the correct size. • Equip the bicycle with proper lights and reflectors. • Have the bicycle inspected to ensure good mechanical condition. • When riding as a passenger, wear appropriate-size helmet and sit in a specially designed protective seat. Wilson, D., Hockenberry, M. (102018). Wong's Nursing Care of Infants and Children, 11th Edition. Last Updated - 11, Oct 2022

Which of the following statements about carbon monoxide is accurate? A. Carbon monoxide is a gas that is gray in color and deadly. B. Carbon monoxide is a gas that is clear, odorless, and deadly. C. Carbon monoxide is a gas that is yellow and odorless. D. Carbon monoxide is a gas that smells like rotten eggs.

Explanation Choice B is correct. Carbon monoxide is a gas that is clear, odorless, and deadly. This invisible gas can build up in enclosed areas where engines, such as car engines, or kerosene heaters, are running idly. Choice A is incorrect. Carbon monoxide is deadly, but it is not gray. Choice C is incorrect. Carbon monoxide is odorless, but it is not yellow. Choice D is incorrect. Carbon monoxide is odorless. Hydrogen sulfide is a gas commonly described as smelling similar to rotten eggs. Learning Objective Recognize that carbon monoxide is a clear, odorless, and deadly gas. Additional Info Carbon monoxide poisoning is one of the most common types of fatal poisoning. Consider toxicity in clients with nonspecific symptoms or unexplained metabolic acidosis. Do not rule out toxicity based on a normal carbon monoxide level because levels can decrease rapidly, particularly after treatment with supplemental oxygen. Treat carbon monoxide poisoning clients with 100% oxygen. For severe poisoning, consider treatment with hyperbaric oxygen. Last Updated - 11, Jan 2022

The occupational health nurse is conducting an in-service on reducing back injuries. It would be correct for the nurse to identify the most common location of the injury is the A. cervical spine. B. lumbar spine. C. thoracic spine. D. pelvis.

Explanation Choice B is correct. The most common area injured during lifting is the lumbar spine. This is because it supports the lower back. Choices A, C, and D are incorrect. These options are incorrect as the lumbar spine is most often the target of injuries while lifting. Additional Info Effective Measures to Prevent Back Injury Include Have the necessary assistance to move the object. Planning the move and communicating with the other individual who will assist you. Using the shoulder, upper arms, hips, and thighs as the predominant muscles to help with the move. Keep objects close to your body when lifting or carrying objects. Avoid twisting by using your feet to turn your body. Use a mechanical lift when necessary. Last Updated - 21, Aug 2022

A health care provider (HCP) orders the immediate use of a piece of electrical care equipment for a client. When you go to use the piece of equipment, you immediately suspect it may be faulty. Your initial action should be which of the following? A. Try the piece of electrical care equipment and see if it becomes hazardous. B. Call the health care provider and report your suspicion. C. Ask the client if they want you to try the piece of electrical care equipment. D. Immediately remove the piece of electrical care equipment from service.

Explanation Choice D is correct. If you suspect that a piece of electrical care equipment may be faulty and you have a health care provider's (HCP) order for the immediate use of this equipment for the client, you should immediately remove it from service. You must not use this piece of equipment under any circumstances. Choice A is incorrect. If you suspect that a piece of electrical care equipment may be faulty and you have a health care provider's (HCP) order for the immediate use of this equipment for the client, you would not "[t]ry the piece of electrical care equipment and see if it becomes hazardous" because the practice would be considered dangerous, reckless, and against the standard of care. Choice B is incorrect. If you suspect that a piece of electrical care equipment may be faulty and you have a health care provider's (HCP) order for the immediate use of this equipment for the client, your initial action would not be to call the HCP. Choice C is incorrect. If you suspect that a piece of electrical care equipment may be faulty and you have a health care provider's (HCP) order for the immediate use of this equipment for the client, you would not ask the client if they want you "to try the piece of electrical care equipment," because the practice would be considered dangerous, reckless, and against the standard of care. Learning Objective When identifying a suspected piece of faulty electric care equipment, prioritize the removal of the piece of equipment. Additional Info Immediately remove any suspected faulty equipment that may be faulty and send it for repairs by an authorized technician or engineer.

The nurse is caring for a client with a sacral wound infected with Methicillin-resistant staphylococcus aureus. Which personal protective equipment (PPE) is necessary to care for this client? Select all that apply. A. Gloves B. N95 respirator C. Surgical Mask D. Goggles E. Gown

Explanation Choices A and E are correct. A gown and gloves should be used when coming into contact with an MRSA wound. This prevents secretions from the wound from infecting the nurse. Choices B, C, and D are incorrect. MRSA in the wound requires contact precautions. A mask is not necessary, nor is goggles or a respirator. Additional Info MRSA is a gram-positive bacteria that is found frequently in healthcare facilities. MRSA is spread by direct contact and affects most older adults through indwelling urinary catheters, vascular access devices, open wounds, and endotracheal tubes. It is susceptible to only a few antibiotics, such as IV vancomycin and oral linezolid. For a client on contact precautions, the door may remain open. During client transport, the wound should be covered with a dry dressing.

The home health nurse is discussing environmental safety with a 74-year-old patient who lives with her son. Which of the following statements by the patient would indicate that additional teaching is needed? A. "My son will install grab bars in the bathroom." B. "I will wear my indoor shoes while walking inside the house." C. "The furniture is arranged so that I can hold onto something if I need it." D. "We will remove all small rugs." Submit Answer

Explanation Choice C is correct. Furniture should be arranged so that there are clear paths, free of rugs, cords, or other obstacles. It is not safe for the patient to be using furniture for support during walking. The nurse should discuss the risks associated with this action and evaluate the patient's need for a mobility aid such as a walker or cane. Choice A is incorrect. Falls frequently occur in the bathroom setting. Grab bars, elevated toilet seats, and shower chairs are examples of safety precautions to reduce the risk of falls. Choice B is incorrect. The patient should wear sturdy, properly fitting footwear when ambulating, even when inside the home. Choice D is incorrect. Rugs should be taped down at the edges or removed from the floors to reduce the risk of falls. NCSBN Client need Topic: Safety, Subtopic: Home safety Last Updated - 06, Nov 2021

The patient just arrived from the operating room after the hypophysectomy surgery was performed. In order to reduce the possibility of surgical complications, which position is the best option for this patient? A. Trendelenburg B. Side-lying C. Semi-fowler's to Fowler's D. Reverse Trendelenburg Submit Answer

Explanation Choice C is correct. Hypophysectomy is generally performed via the transsphenoidal route to remove tumors from the pituitary gland. Semi-Fowler's to Fowler's position is the most appropriate position as it facilitates drainage and prevents swelling to the head and neck or an increase in intracranial pressure. Choice A is incorrect. Trendelenburg would be a precarious position in this patient, increasing intracranial pressure and creating swelling. Choice B is incorrect. Side-lying does not promote draining, which will be needed in this patient's care. Choice D is incorrect. Reverse Trendelenburg is too drastic of a position for this patient. NCSBN client need Topic: Reduction of Risk Potential: Surgical Complications and Health Alterations Last Updated - 15, Nov 2022

The nurse in the emergency department is preparing to receive a client exposed to inhalation anthrax. The nurse plans to implement A. droplet precautions. B. airborne precautions. C. standard precautions. D. contact precautions. Submit Answer

Explanation Choice C is correct. Standard precautions are utilized in the management of inhalation anthrax. Inhalation anthrax is not transmitted from person to person, and its vector is contaminated materials, such as wool, hides, or hair. Choices A, B, and D are incorrect. Inhalation anthrax does not spread from person to person, and implementing contact, droplet, or airborne precautions would be unnecessary. Additional Info Anthrax is a bioterrorism agent and must be taken seriously because it has a high mortality rate. Anthrax may be cutaneous or inhaled and is caused by exposure to the gram-positive bacterium. Nursing care is aimed at stabilizing the client's breathing and promptly initiating treatment, which is antibiotics (levofloxacin) and/or antitoxins such as raxibacumab.

A nurse is reinforcing education regarding home fire safety to the parents of a 5-year-old. Which of the following would be accurate to include in the teaching plan? Select all that apply. A. Most fatal fires occur when people are cooking. B. Most people who die in fires die of smoke inhalation rather than burns. C. Less than 60% of U.S. fire deaths occur in the home. D. About 60% of fire deaths occur in a home without a smoke detector. E. More fires occur in homes occupied by single parents. F. In the event of a fire, quickly call the fire department and then exit home. Submit Answer

Explanation Choices B and D are correct. Most people who die in home fires indeed die of smoke inhalation rather than burns (Choice B). Smoke inhalation is the most common cause of death in a house fire. Symptoms of smoke inhalation include coughing, nausea, vomiting, drowsiness, and confusion. If a fire victim has difficulty breathing, singed nostril hairs, and burns to their nose, mouth, or face, then immediate medical attention is warranted because this may indicate the individual has inhaled scorching air, which can cause burns in the respiratory tract. More than 50 percent of people with severe burns and smoke inhalation die. Fire, while burning, consumes the oxygen in an enclosed space leaving little oxygen for occupants to breathe and maintain consciousness. Lack of oxygen may cause impaired consciousness quickly and occupants are often not able to reach the exits. In addition to this, toxic gases like carbon monoxide replace the oxygen and can be fatal even in small amounts. Fire experts often recommend conducting a family fire drill twice a year, which may include practicing to get out of the house while blindfolded. The family should be taught to exit first since time is of the essence during a fire, and exiting should not be delayed searching for a phone or calling a neighbor/fire department for help. Such calls should be made after exiting. Studies have found that about 60% of home fire deaths occur in a home without a smoke detector. So, a nurse needs to reinforce the education regarding smoke alarms and the client should make sure these alarms are working. Choice A is incorrect. Most fatal fires occur when people are sleeping, not while cooking. Choice C is incorrect. it is not less than 60%. 85% of fire-related deaths occur in the home, not in public places. Choice E is incorrect. Being a single parent does not increase the risk of experiencing a fire in the home. Choice F is incorrect. In the event of a fire, first, exit home and then call for help from the fire department or neighbor. Since smoke can quickly overcome an occupant, time is of the essence, and the occupant should exit the home STAT. NCSBN Client Need Topic: Safety and Infection Control; Subtopic: Home safety Last Updated - 14, Feb 2022

The nurse is caring for assigned clients. Which of the following clients should the nurse identify is at the highest risk for falling? A. 88-year-old admitted with a chest tube secondary to pneumothorax and has a history of dementia. B. 44-year-old admitted with heart failure, has a peripheral IV, and receiving IV furosemide. C. 33-year-old admitted with cholecystitis, has a peripheral IV, and is receiving IV hydromorphone. D. 28-year-old admitted with bacteremia, is receiving intravenous fluids via central line, and is diaphoretic. Submit Answer

Explanation Choice A is correct. This client has advanced age, has a medical device that impedes their mobility, and has cognitive impairments. Thus, all these risk factors make this client at a very high risk for falls. Choices B, C, and D are incorrect. All these clients have risk factors for falls; however, none of these clients have advanced age or have a cognitive impairment. Thus, universal fall precautions should be instituted, but it is the 88-year-old client who requires aggressive fall reduction measures. Additional Info Risk factors for falls include Advanced age Cognitive impairments (delirium, dementia) History of previous falls Medical/Assistive device(s) (chest tube, peripheral IV, cane) Medications (anticholinergics, benzodiazepines) Urinary frequency Universal fall precautions include Hourly rounding that addresses needs such as toileting Thorough room orientation and frequent reminders Adequately lit room with appropriate markings Accessible call light that is within reach Pathways clear of clutter and grab bars in the bathroom Additional measures that may be taken include relocating the client closer to the nursing station, enhanced observation, and using a bed alarm. Last Updated - 15, Feb 2022

The nurse is participating in a committee reviewing strategies to reduce falls in the older adult. Which of the following recommendations by the nurse would be appropriate to make? A. Increase the number of bedside commodes in the nursing units B. Provide more hand sanitizer stations in high traffic areas C. Standardize administration times of diuretics to the evening hours D. Implement a bedside handoff reporting process for nursing staff

Explanation Choice A is correct. Utilizing more bedside commodes for older adults may reduce falls because it shortens the distance a client needs to travel to the bathroom. Bedside commodes are especially effective for those receiving medications such as diuretics and undergoing bowel prep. Thus, a bedside commode is an effective intervention to reduce falls for the older adult. Choices B, C, and D are incorrect. Increasing the number of hand sanitizer stations would not reduce falls as this is an effective intervention for reducing infections. Standardizing administration times of antihypertensive and diuretics would be helpful, but the times should not be standardized to the evening hours. Diuretics should be administered early in the day to prevent nocturia. Handoff reporting for nursing staff at the bedside is an effective tool to increase client satisfaction and communication but not a mechanism to reduce falls. Since handoff reporting happens at a predictable interval and is infrequent, this would not be the most effective intervention to recommend to mitigate falls. Learning Objective NCLEX Category: Safety and Infection Control Related Content: Accident/Error/Injury Preventionn Question type: Application Additional Info Universal fall precautions involve • Monitor the client's activities and behavior as often as possible, preferably every 30 to 60 minutes. • Teach the client and family about the fall prevention program to become safety partners. • Remind the client to call for help before getting out of bed or a chair. • Help the client get out of bed or a chair if needed; lock all equipment such as beds and wheelchairs before transferring client's. • Teach clients to use the grab bars when walking in the hall without assistive devices or when using the bathroom. • Provide or remind the client to use a walker or cane for ambulating if needed; teach him or her how to use these devices. • Remind the client to wear eyeglasses or hearing aid if needed. Last Updated - 17, Apr 2022

The nurse is observing a newly hired nurse apply bilateral wrist restraints to a client. Which action by the newly hired nurse requires follow-up? A. Secures the restraint to the frame of the bed. B. Repositions the client from semi-Fowlers to prone. C. Provides easy access to the quick release buckle. D. Assesses the radial pulse every two hours. Submit Answer

Explanation Choice B is correct This action is not appropriate and requires follow-up. A client in physical restraints should not be positioned prone, which may lead to suffocation. Additionally, a client should not be positioned supine because this makes the client feel vulnerable. Choices A, C and D are incorrect. These actions by the newly hired nurse are appropriate and do not require follow-up. Physical restraints should not be secured to the side rails as this may result in physical injury to the client. Securing the restraint with a knot is no longer acceptable clinical practice and the nurse should engage a quick release buckle that is anchored to the bed frame. Part of the assessment of a client in bilateral wrist restraints includes the client's radial pulses to determine the client's neurovascular status. Additional Info Restraints should be used as a last resort if alternative methods are not effective. A nurse should never threaten a client with restraints. This is considered assault. The nurse may place a client who is violent in restraints without an order from the primary healthcare provider (PHCP). If this was to occur, the nurse has one hour to inform the provider and obtain an order. Restraints are never as needed (PRN). They should be discontinued at the earliest possible time. When restraining a client, the reason for the restraint must be explained to the client and the behavior the client needs to demonstrate for the restraints to be discontinued. The nurse should observe the client at frequent intervals to offer nutrition & toileting, assess their behavioral status, obtain vital signs, and provide range of motion. These intervals are determined by the facility and the type of restraint—the more restrictive the restraint and the younger the client, the more frequent assessment. Restraints must be able to quickly be removed via a quick release buckle (knots are no longer recommended). The nurses' documentation must be comprehensive, describing the reasoning for the restraints, alternatives utilized, the education provided to the client, the type of restraint utilized, how it was secured, and the ongoing behavior necessary to continue the restraint. The nurse should also document the intervals at which the restraints were released. Last Updated - 01, Jun 2022

The nurse is discussing infection control with a group of nursing students. It would be correct to state that airborne precautions are used for which condition? Select all that apply. A. Pulmonary tuberculosis B. Pertussis C. Rubeola D. Hepatitis A E. Rubella Submit Answer

Explanation Choices A and C are correct. Conditions requiring airborne precautions include pulmonary tuberculosis and rubeola. Choices B and E are incorrect. Pertussis and rubella are diseases requiring droplet precautions. Choice D is incorrect. Hepatitis A is managed with standard precautions.

There is a massive airline crash near your acute care facility. As the victims of this massive external disaster arrive at your facility, your new graduate nurse asks you what the black-colored triage tags on the incoming victims indicate. How should you respond to this new nurse? A. The victims are the lowest priority for care. B. The victims have life-threatening injuries and are in need of immediate care. C. The victims are always dead. D. The victims are in a severe medical crisis, and they have little chance of survival. Submit Answer

Explanation Choice D is correct. You should tell the new graduate nurse that the black-colored triage tags on the incoming victims mean that the victims are in a severe medical crisis, and they have little chance of survival. In mass casualty scenarios, an advanced triage system is implemented and involves a color-coding scheme using red, yellow, green, white, and black tags. Remember the "DIME" acronym: Delayed; Immediate, Minor, Expectant. Red tags- IMMEDIATE - highest priority treatment/transfer. These patients cannot survive without immediate treatment but have a high chance of post-treatment survival. E.g. Tension pneumothorax, cardiac tamponade, massive hemorrhage. Yellow tags- DELAYED - medium priority. No immediate danger of death, stable but will still need hospital care. Under normal circumstances, these patients will be treated immediately, but in mass casualty scenarios, they are medium priority. E.g. isolated humerus or femur fracture. Green Tags- MINOR - lowest priority - those with minor injuries, ambulating ("walking" wounded). E.g. abrasions sprain. These are attended to after high and medium-priority patients are addressed. Black Tags- EXPECTANT - keep comfortable, pain medications only until death. These are patients with injuries so extensive that they will not be able to survive with the best available care or those dead already. E.g. Massive head injury with fixed pupil, third-degree burns involving 95% body surface. Walking away from those with black tags can be emotionally and ethically challenging. About "expectant" victims, WHO states, "It is unethical for a physician to persist, at all costs, at maintaining the life of a patient beyond hope, thereby wasting to no avail scarce resources needed elsewhere." Choice A is incorrect. You should not tell the student nurse that black-colored tags on the incoming victims mean that the victims are the lowest priority for care. Green indicates that the victims are the lowest priority for care. Black has no preference for ongoing care, and only comfort care needs to be provided. Choice B is incorrect. You should not tell the new graduate nurse that the black-colored triage tags on the incoming victims mean that the victims with these black tags have life-threatening injuries and need immediate care. The red color tag indicates that the victims have fatal injuries and need urgent attention. Choice C is incorrect. You should not tell the new graduate nurse that the black-colored triage tags on the incoming victims always means that the victims are dead because it could also represent critically injured but unsalvageable victims. Last Updated - 02, Feb 2022

The emergency department (ED) charge nurse is preparing for a surge of clients diagnosed with Ebola virus disease (EVD). The nurse should plan to take which action? Select all that apply. A. Implement visitor restrictions for affected clients B. Log entry and exit of all healthcare workers who provide care C. Ensure that bleach disinfectant wipes are available in each room D. Provide reusable personal protective equipment E. Have an observer for donning and doffing of personal protective equipment Submit Answer

Explanation Choices A, B, C, and E are correct. EVD is highly contagious, and despite a vaccine being available, it has a high mortality rate. For any infectious outbreak, visitation should be restricted. This also includes non-essential healthcare workers. When caring for a client with a pathogen like Ebola, logging the entry and exit of healthcare providers would be helpful to contact trace for any potential exposures. Bleach is the cleaning agent of choice, and bleach sprays and wipes should be readily available. Having a trained observer while staff don and doff PPE has effectively prevented accidental exposures. Choice D is incorrect. Single-use PPE should be utilized - not reusable. Reusable PPE would substantially raise the risk of exposing individuals to the virus. Additional Info Ebola virus disease (EVD) Standard, contact, and droplet precautions should be utilized. If aerosolized procedures should be performed (nebulizer, bronchoscopy), airborne precautions should be implemented. Do not cohort a client with ebola with others Required PPE Single-use (disposable) fluid-resistant gown that extends to at least mid-calf or single-use (disposable) fluid-resistant coverall without integrated hood. Single-use (disposable) full-face shield. Single-use (disposable) surgical mask. Single-use (disposable) gloves with extended cuffs. Two pairs of gloves should be worn. At a minimum, outer gloves should have extended cuffs. Single-use (disposable) ankle-high shoe covers. Shoe covers should allow for ease of movement and not present a slip hazard to the wearer. Single-use (disposable) powered air-purifying respirators (PAPR) or N95 mask is recommended if aerosolized procedures are performed. Last Updated - 07, Jul 2022

The nurse is educating a client about the inactivated influenza vaccine (IIV). The nurse should plan to teach the client that Select all that apply. A. the IIV effectively prevents influenza or decreases the disease's severity. B. pregnant women can receive this vaccine. C. you may receive this vaccine if you are allergic to penicillin. D. the IIV contains a live virus. E. the vaccine is administered to newborns following delivery. Submit Answer

Explanation Choices A, B, and C are correct. Although the influenza vaccine will not prevent 100% of the cases, it will help to prevent or decrease symptoms in 70 to 80% of the cases. The influenza vaccine is recommended for all pregnant women and is safe for this population. Individuals on antibiotics for a mild or moderate illness can receive the influenza vaccine. These individuals can receive any immunization. Penicillin allergy is not a contraindication to the administration of influenza. Choice D is incorrect. The IIV is inactivated and does not contain a live virus. IIV is administered to infants starting at six months. Thus, the infant's caregivers must get vaccinated to prevent transmission to the newborn. Additional Info ➢ The influenza vaccine may be administered to those six months and older. ➢ Contraindications to vaccine administration include anaphylaxis to previous doses. Egg allergy is not an absolute contraindication to vaccine administration. ➢ A history of Guillain-Barré syndrome within 6 weeks of receiving an influenza vaccine is another contraindication to all influenza vaccines. ➢ Influenza vaccine may be coadministered with another vaccine such as COVID-19. ➢ IIV and LAIV are available. LAIV (live attenuated influenza vaccine) is administered via nasal mist. This vaccine is recommended for individuals who are immunocompetent aged 2 through 49. Last Updated - 02, Dec 2022

While working in an outpatient clinic, you take vital signs for a woman who expresses her interest in using herbal therapies to treat her chronic back pain. As a nurse, you know that herbal therapies can be safe when used properly, but should be closely monitored. You review the following teaching points with her to ensure her safe use of any herbal therapies. Select all that apply. A. Tell your health care provider about any herbal therapies you are using. B. Only take the recommended dose of the herbal therapy to avoid any toxicity. C. Continue taking your prescribed medications from your healthcare provider; never stop taking a medication without talking to your health care provider. D. Using herbal remedies is acceptable for any condition as long as they are supervised by a healthcare provider. Submit Answer

Explanation Choices A, B, and C are correct. The nurse should teach the patient to tell her healthcare provider about any and all herbal therapies she is using. It is important for the patient to understand that these therapies should be treated as seriously as any medication and that her healthcare provider will need to know everything she is taking to prevent any side effects or adverse reactions. Herbal therapies used in doses higher than what is recommended can quickly become toxic and the client should be instructed on this. Discontinuing a prescribed medication, even if adding an herbal therapy, is never recommended and could be dangerous. The client should be educated never to do this. Choice D is incorrect. Herbal remedies are not appropriate for all conditions. For example, in serious medical conditions such as heart disease or stroke, herbal remedies are not appropriate treatments. NCSBN Client Need: Topic: Psychosocial Integrity Subtopic: Cultural Awareness/Cultural Influences on Health Last Updated - 02, Dec 2021

The charge nurse is observing infection control practices in the nursing unit. Which observation by the charge nurse requires follow-up? Select all that apply. A. Soiled linens are placed on the floor during a bed bath and linen change. B. A disposable blood pressure cuff is used for a client on contact precautions. C. Disposable dishes are placed in the room for a client on droplet precautions. D. Reusable eye protection is cleaned and disinfected after each client encounter. E. A surgical hand scrub is performed with the hands lower than the elbows. F. Gloves are doffed last while removing personal protective equipment (PPE).

Explanation Choices A, C, E, and F are correct. If the charge nurse observes these actions, they require follow-up because they are incorrect. Soiled linens should not be placed on the floor because they contaminate other surfaces in the facility. Once soiled linens are removed, they should go in the appropriate fluid-resistant containers. Disposable dishware and utensils are not required for clients on transmission-based precautions. The hot water and detergents used in dishwashers are sufficient to decontaminate dishes and eating utensils, so the client should be provided reusable, washable dishware. If a surgical hand scrub is being performed, the appropriate technique is to avoid contamination by holding hands above the elbows as fluid flows in the direction of gravity. Gloves are removed first during the doffing of PPE. The sequence for doffing (removing PPE) is gloves, face shield or goggles, gown, and mask or respirator. Choices B and D are incorrect. These observations do not require follow-up because they adhere to appropriate practices. Disposable equipment should be utilized for a client on contact precautions. This equipment includes thermometers and blood pressure cuffs. Reusable eye protection may be disinfected and cleaned after each client encounter. The eye protection should be allowed to dry before its next use. Additional Info The nurse can prevent disease transmission by exercising meticulous hand hygiene, educating others, and correcting inappropriate actions. The appropriate sequence of donning of PPE is gown, mask or respirator, goggles or face shield, and gloves. The appropriate sequence for doffing PPE is gloves, face shield or goggles, gown, and mask or respirator. Last Updated - 11, Apr 2022

The nurse is demonstrating the appropriate use of a car seat to a client. The nurse is demonstrating which level of prevention? A. Primary B. Secondary C. Tertiary D. Quaternary

Explanation Choice A is correct. Primary prevention is often referred to as the true level of prevention because it occurs before disease or illness. Demonstrating the appropriate use of a car seat is primary prevention because it happens before an automobile crash, a leading cause of death for those younger than 19. Choices B, C, and D are incorrect. Demonstrating the use of the car seat before an automobile crash is a primary level of prevention. Please see the additional information section for examples of the other levels of prevention. Additional Info Last Updated - 24, Aug 2022

Which of the following is an internal disaster? Select all that apply. A. A loss of electrical power to the facility B. The sudden cessation of internal communication C. A toxic chemical spill in the lobby of the facility D. A serious life threatening medication error Submit Answer

Explanation Choices A, B, and C are correct. A loss of electrical power to the facility, the sudden cessation of internal communication, and a toxic chemical spill in the lobby of the facility are all examples of domestic disasters. Other cases of civil emergencies include things like a fire, a bomb threat, a cyclone, a flood, a tornado or hurricane that affects the healthcare facility. Choice D is incorrect. A medication error is not considered an internal disaster or an external disaster. Last Updated - 20, Aug 2021

A nurse is caring for a client with pneumonia who is in bilateral wrist restraints. The client has developed confusion. The nurse should take which priority action? A. Obtain vital signs B. Release restraints and provide range of motion C. Auscultate lung sounds D. Assess skin integrity under each restraint

Explanation Choice A is correct. A complication of pneumonia is acute respiratory distress syndrome (ARDS). The hallmark of ARDS is hypoxemia which may manifest as altered mental status. The nurse should obtain vital signs with an emphasis on assessing the client's respiratory rate and pulse oximetry. Choices B, C, and D are incorrect. The restraints inhibit the client from mobilizing, which is detrimental to the management of pneumonia. However, the nurse should immediately obtain vital signs to ascertain if hypoxia is causing this confusion. Auscultating lung sounds is not an essential action. This would not generate useful clinical data as it cannot reveal if the client is hypoxic. Assessing the skin integrity of the wrists and providing range of motion are not actions that would remedy the client's confusion. Additional Info Acute respiratory distress syndrome (ARDS) has clinical features such as: • Hypoxemia that persists even when 100% oxygen is given (refractory hypoxemia, a cardinal feature) • Decreased pulmonary compliance • Dyspnea • Non-cardiac-associated bilateral pulmonary edema • Dense pulmonary infiltrates on x-ray (ground-glass appearance) Measures that prevent a client with pneumonia from developing ARDS include aggressive mobilization, incentive spirometry, prescribed bronchodilators, and prompt initiation of antibiotics.

The nurse is caring for a client who has been physically violent towards staff. The nurse prepares to restrain the client using A. soft wrist restraints. B. mitten restraints. C. elbow restraints. D. waist belt restraint. Submit Answer

Explanation Choice A is correct For the client being physically violent towards staff, the nurse may, as prescribed, chemically restrain the client or physically restrain the client. The appropriate option for this client is to either seclude the individual or use soft wrist restraints. Choices B, C, and D are incorrect. Mitten restraints would be appropriate if the client attempted to disconnect medical tubing or devices. This could be plausible if tethered to the frame, but mittens are more restrictive than necessary. Elbow restraints make removing a medical device near the face or neck difficult. It does not impede the removal of abdominal or urinary medical devices. Waist belt restraints would still allow the client to hit staff. This type of restraint is best utilized for confused or impulsive clients who are continually trying to get out of bed or a chair after repeated redirection. Additional Info Soft wrist restraints are best utilized for clients becoming increasingly agitated, cannot be redirected with distraction, and keep trying to remove needed medical devices. This type of restraint may also be applied to the ankles. Elbow restraints make it difficult for a client to remove a medical device near the face or neck. It does not impede the removal of abdominal or urinary medical devices. Waist belt restraints are utilized for confused or impulsive clients who continually try to get out of bed or a chair after repeated redirection. Mitten restraints are only considered a restraint if they are tethered to an immovable object, and the client cannot remove it from their hand. This restraint is appropriate if the client attempts to disconnect medical tubing or devices. Last Updated - 01, Jun 2022

A 16-year old male arrives at the trauma bay after suffering from a head injury. He is postictal and is being intubated to protect his airway. After obtaining IV access, the doctor orders a diuretic. Based on the history provided, what diuretic will the doctor order? A. Mannitol B. Hydrochlorothiazide C. Spironolactone D. Furosemide

Explanation Choice A is correct. Mannitol is the preferred diuretic used for reducing the increased intracranial pressure. Mannitol is an osmotic diuretic that does not cross the blood-brain barrier. As a result; osmotic pressure increases in the plasma but not in the brain. A gradient is developed between plasma and brain cells, therefore causing a shift of fluids from the extracellular space into the blood vessels. Choice B is incorrect. Hydrochlorothiazide ( HCTZ) is a thiazide diuretic. HCTZ is used in the treatment of hypertension and peripheral edema. Choice C is incorrect. Spironolactone is used to treat high blood pressure, ascites, edema, and to antagonize high levels of hormones ( aldosterone) in hyper-aldosteronism. Choice D is incorrect. Furosemide is a loop diuretic. It helps decrease edema and fluid retention caused by congestive heart failure/ heart disease, liver disease, and kidney disease. NCSBN Client Need Topic: Pharmacological and Parenteral Therapies, Subtopic: Acute Intracranial Problems Last Updated - 11, Jan 2022

The nurse in the emergency department (ED) is caring for a child with erythema infectiosum (Fifth disease). Which transmission-based precautions should the nurse implement? A. Standard B. Droplet C. Contact D. Airborne Submit Answer

Explanation Choice A is correct. Standard precautions are used for a client with erythema infectiosum. Droplet precautions would only be necessary if the client is immunocompromised. Choices B, C, and D are incorrect. Isolation is not necessary for this virus. The exception to this would be if the client was immunocompromised which would then require standard + droplet. Additional Info Erythema infectiosum (Fifth disease) The causative agent is Parvovirus B19 Mode of transmission for this pathogen is respiratory secretions and blood, blood products Isolation is not necessary unless the child is immunosuppressed (standard/droplet will then be used) Manifestations include erythema on the face (slapped face appearance). It also causes the appearance of maculopapular red spots distributed on the upper and lower extremities. Finally, the client will have mild flu-like symptoms such as a fever, headache, and malaise Treatment is primarily supportive (antipyretics and analgesics) A blood transfusion may be needed for transient aplastic anemia Last Updated - 08, Dec 2022

Which of the following statements about security in healthcare environments is accurate? A. Healthcare facilities must have egress alarms on all doors, except client doors, to maintain security within the facility. B. All members of the healthcare facility must have education and training relating to security in the facility. C. Members of the healthcare facility who do not have clinical access do not need education and training relating to security in the facility. D. Members of the healthcare facility who have only clerical roles do not need education and training relating to security in the facility.

Explanation Choice B is correct. All healthcare facility members must have education and training relating to security in the facility. Choice A is incorrect. Healthcare facilities are not mandated to have egress alarms on all doors, except client doors, to maintain security within the facility. Egress alarms are, however, prescribed for high-security risk areas. Choice C is incorrect. Members of the healthcare facility who do not have clinical access are required to have education and training relating to security in the facility because all areas and all departments are at risk for security breaches. Choice D is incorrect. Members of the healthcare facility who have only clerical roles are required to have education and training relating to security in the facility because all areas and all departments are at risk for security breaches. Last Updated - 26, Jan 2022

The nurse is caring for a client who repeatedly attempts to get up from their wheelchair unassisted and has fallen twice. The primary healthcare provider (PHCP) prescribes restraints. Which type of restraint does the nurse anticipate? A. Soft wrist restraints B. Mitten restraints C. Seclusion D. Waist belt restraint

Explanation Choice D is correct If restraints are necessary for this client, the most appropriate restraint would be a waist belt restraint. This restrains the client's ability to stand up, which is the device's intent. Choices A, B, and C are incorrect. Soft wrist restraints would not be appropriate for this clinical situation. This type of restraint would be appropriate for clients who pull at tubes or devices. Soft wrist restraints may also be suitable for a client who is agitated and has hit themselves or others. Mitten restraints may be indicated for a client who pulls at tubing or devices. Seclusion is a form of restraint and would be contraindicated in this case as seclusion is used for individuals who are violent. Clients who are at risk of falling should not be placed in seclusion. Additional Info Belt restraints may be warranted for confused or impulsive clients who are continually trying to get out of bed or a chair after repeated redirection when it's unsafe for them to get up unassisted. Belt restraints should be applied over clothing and secured over the client's waist. The restraint should be anchored to an immovable part of the bed or the chair. Last Updated - 01, Jun 2022

The nurse is teaching a group of students about incident reports. Which of the following statements, if made by the student, would require further teaching? Select all that apply. A. "Reporting can only be completed if it is within one hour after the event." B. "Witnesses to an incident should be mentioned in the report." C. "A client eloping does not require an incident report." D. "A slip and fall by a client should be reported." E. "Incidents involving visitors do not have to be reported." Submit Answer

Explanation Choices A, C, and E are correct. Incident (sometimes termed occurrence or event) reporting is required when any type of activity deviates from the norm. This could include a fall, medication error, elopement, unplanned transfer of care, client complaint, or a delay in care. It is highly recommended that the event be reported after it occurs to ensure accuracy; however, event reporting should be completed as soon as possible, including the next day. No rule exists stating that reports must be completed within one hour after the actual event. Incidents involving visitors such as a fall, misconduct, complaint, or injury should be reported. Choices B and D are incorrect. The incident report should be completed with cohesion and based on facts. Supporting statements from witnesses and the identification of witnesses are appropriate to include in the report. Additional Info Incident (sometimes termed occurrence or event) reporting is a tool to mitigate future risks. Incident reporting should also be completed for events involving clients and visitors. Such events include: Verbal and physical displays of aggression Tampering with medical devices Reports of sexual or physical abuse by staff Adverse reaction to a blood transfusion Client elopement Damage (or loss) of client possessions Falls or injuries Injuries related to a medical device Complaints Medication and treatment errors Interfering with client care The incident should not be logged in the medical record or nursing notes. The documentation should be objective and factual and include what occurred, any injuries, if the provider was notified, care administered after the event, and any witnesses. Last Updated - 17, Sep 2022

The school nurse is attending to a student who got a chemical cleaner in her eyes. In which order should the following actions be performed? Document the occurrence Irrigate the eye from the inner canthus to the outer canthus Check the pH of the eye Call the child's parent Assess the visual acuity Submit Answer

Explanation Ocular chemical injuries (alkaline or acidic substances) can cause permanent damage and vision loss. Therefore, a chemical eye injury is considered a medical emergency. When a chemical injury is sustained, the school nurse should check the pH and immediately irrigate the student's eye. Sterile water or 0.9% saline can be used for eye irrigation. In an emergency, tap water can be used initially. The steps involved in eye irrigation include: Check the pH of the eye (check pH of both eyes. Alkaline substances penetrate the ocular surface more and cause more damage than acids). If the pH testing is not immediately available, this should not delay irrigation. Get the infusion set ready to regulate the flow of irrigation fluid. Position the patient with the cheek of the affected side pressed against a kidney dish. Irrigate the eye from the inner canthus to the outer canthus. This direction is used so that the chemical does not get into the other eye. Assess the child's visual acuity. Visual acuity testing should not precede eye irrigation since delaying the treatment is dangerous in chemical eye injury. Document the occurrence and the procedure. This documentation should include pH readings, the amount of irrigation fluid used, and the duration of the procedure. Call the child's parents to inform them of the occurrence, actions taken, and the outcome. A procedure note regarding the irrigation should be documented first. Following this, the nurse can call the parents and document the verbal/phone call communication in a separate note. NCSBN client need: Topic: Physiological integrity, physiological adaptation Learning Objective Understand that a chemical eye injury is a medical emergency because it can lead to permanent visual loss. The nurse should check the pH to determine the nature of the contaminant, immediately irrigate the eye with sterile water, and then only proceed with the visual acuity testing.

The nurse is admitting a 72-year-old patient hospitalized for a medical diagnosis of Mycoplasma pneumonia. Which transmission-based precaution is necessary? A. Private room with negative pressure airflow B. Wearing a surgical mask within 3 feet of the patient C. Wearing gloves when in contact with the patient D. HEPA filtration for incoming air Submit Answer

Explanation Choice B is correct. Droplet precautions are indicated for patients with Mycoplasma pneumonia. Droplet precautions include wearing a surgical mask when within 3 feet of the patient, proper hand hygiene, and placement in a private room or with a cohort of patients. Other examples where droplet precautions are indicated include Pertussis, Influenza, Diphtheria, and invasive Neisseria meningitides. There are three types of transmission-based precautions: The model used depends on the mode of transmission of a specific disease. Airborne Contact Droplet Choice A is incorrect. A private room with negative pressure airflow is a component of airborne precautions. Airborne precautions would be indicated for diseases such as Measles, Severe Acute Respiratory Syndrome (SARS), Varicella (chickenpox), and Mycobacterium tuberculosis. Choice C is incorrect. Wearing gloves when in contact with the patient is required with contact precautions. Presence of diarrhea/stool incontinence (i.e. Norovirus, Rotavirus, Clostridium difficile), or skin infections (i.e. MRSA, Vancomycin-Resistant Enterococci (VRE)), open or draining wounds, pressure ulcers, generalized rash, or presence of ostomy tubes and bags draining body fluids. Choice D is incorrect. HEPA filtration for air coming into the patient's room is associated with a protective environment. Last Updated - 16, Jan 2022

The nurse plans to use a mechanical lift to transfer a client from a stretcher to a wheelchair. Which appropriate action should the nurse take? A. Keep the stretcher's side rails raised during the transfer B. Instruct the client to fold their arms over their chest C. Apply gloves and gown for this procedure D. Unlock the wheels on the stretcher and wheelchair Submit Answer

Explanation Choice B is correct. During any mechanical lift transfer, the nurse should instruct the client to fold their arms over the chest, preventing injuries to the client's arms during the transfer. Choices A, C, and D are incorrect. The side rails of the stretcher should be lowered during the actual transfer because the side rails being raised may impede the client from transferring the stretcher to the wheelchair. If the side rails were raised, this would cause the nurse to raise the client even higher using the lift, which could cause injury if the client were to fall. Gloves and a gown are unnecessary for this procedure and would waste facility resources. Standard precautions are sufficient for this task. The wheels of the stretcher and wheelchair should be locked to prevent the client from slipping from either. Additional Info ✓ A mechanical lift should only be used by staff who have been appropriately trained. ✓ A mechanical lift may be hydraulic or be affixed to the ceiling. ✓ The nurse (or UAP) should always check the weight restrictions for the lift before its use. ✓ When applying a mechanical lift to a client, the lift will be applied with the client lying supine in bed. ✓ A second staff member must assist the hydraulic lift to lower the client into the chair (or back to bed). Last Updated - 06, Feb 2023

The nurse is planning a staff development conference about ways to prevent the transmission of the hepatitis C virus to healthcare workers. It would be appropriate for the nurse to cover which topic? A. How to obtain the HCV vaccine B. How to dispose of sharps safely C. How to dispose of urine and feces for those with HCV D. Isolation precautions for individuals with HCV Submit Answer

Explanation Choice B is correct. The primary way HCV is transmitted is through blood exposure which a needle stick may trigger. The nurse should discuss safety regarding the disposal of needles, such as the importance of not recapping needles. Choices A, C, and D are incorrect. HCV does not currently have a vaccine. A vaccine is available for hepatitis A and B, but it is not available for HCV. However, a robust cure for HCV does exist. HCV is not spread through urine or feces. The primary mode of transmission is through the exposure of the infected individual's blood. Individuals with HCV are not isolated and receive standard precautions. Additional Info ✓ Hepatitis C symptoms have an insidious onset. During the illness, the client may be asymptomatic. ✓ Hepatitis C has an incubation period between 2 weeks and six months. ✓ The virus is spread through infected blood. Thus, hepatitis C may be transmitted via needlesticks, unregulated tattooing, perinatal, and contact with infected blood. ✓ While sexual transmission is possible, the risk is relatively low with sexual contact. ✓ No vaccination exists for hepatitis C; however, robust cures are available for specific genotypes. Last Updated - 23, Jan 2023

The nurse is reviewing a newly hired nurse's understanding of sterile technique. Which statement, if made by the newly hired nurse, would indicate effective understanding? Select all that apply. A. "I should open sterile packages away from my body." B. "If the sterile field gets contaminated, I should dispose of everything and start over." C. "One inch (2.5 cm) border around a sterile drape can be touched with clean fingers." D. "I should apply sterile gloves on my non-dominant hand first." E. "An object placed below my waist is considered contaminated." Submit Answer

Explanation Choices A, B, C, and E are correct. To prevent contamination, packages should be opened away from the body (reaching over the sterile field causes contamination). If anything is contaminated, it should be removed immediately and discarded. Contaminated objects should be kept away from the sterile field and the sterile field (and its objects) should be kept above the waist. The nurse should avoid touching the sterile field as much as possible but is permitted to touch the outer one-inch (2.5 cm) edge of the field, if necessary. Choice D is incorrect. The correct technique for applying sterile gloves is to apply the glove to the dominant hand first. Using the non-dominant hand, pick up the glove for the dominant hand by touching only the "inside" of the glove's cuff and then applying it to the dominant hand. Additional Info Last Updated - 09, Sep 2022

The nurse is teaching a group of unlicensed assistive personnel (UAPs) concepts of client identification. Which situation would require two client identifiers? Select all that apply. A. Providing a meal tray B. Changing bed linens C. Replacing a suction cannister D. Obtaining vital signs E. Providing range of motion exercises Submit Answer

Explanation Choices A, D, and E are correct. Anytime the nurse or unlicensed assistive personnel (UAP) engages directly with the client, two identifiers (name and date of birth) should be asked. This prevents misidentification and mitigates errors related to care delivery. Providing a meal tray will require the identifiers because diets vary by client and are prescribed by the primary healthcare provider (PHCP). Obtaining vital signs requires the two identifiers so the nurse (or UAP) may accurately record these vital signs. Finally, providing range of motion requires two identifiers as it is a task directly involving the client. Choices B and C are incorrect. These tasks do not involve the client, nor is there a risk for client harm when these tasks are executed. They do not require two client identifiers. Additional Info Two reliable client identifiers should be verified for client activities. Acceptable client identifiers include Name Date of Birth An assigned identification number (e.g., medical record number, etc). Telephone number or another person-specific identifier Electronic identification technology coding, such as bar coding or RFID includes two or more person-specific identifiers The nurse should not use identifiers such as the client's room number. Last Updated - 10, Jun 2022

The nurse is assessing a client who is requiring bilateral wrist restraints. Which assessment data is necessary for the nurse to obtain? Select all that apply. A. Previous restraint use B. Skin integrity C. Behavioral status D. Vital signs E. Urinary continence Submit Answer

Explanation Choices B, C, and D are correct. Pertinent assessments necessary prior to initiating restraints include the client's skin integrity. If the client has impaired skin integrity, this may inhibit the type of restraint the nurse may utilize (example, sores on the wrists then wrist restraints should not be used). Behavioral status is necessary to assess because it is the client's behavior that determines the need for the restraint as well as the type of restraint (if the client is verbally deescalated, then physical restraints may not be necessary). Vital signs are essential to obtain because if the client is hypoxic, this may cause altered behavior. Choices A and E are incorrect. Previous restraint use should not influence the nurses' ability to determine the need for restraints. Restraint use is episodic and focuses on the client's current behavioral status at the present time. Urinary continence is not pertinent to restraining a client because toileting is offered at certain intervals based on the restraint and facility protocol. Additional Info Restraints should be used as a last resort if alternative methods are not effective. A nurse should never threaten a client with restraints. This is considered assault. The nurse may place a client who is violent in restraints without an order from the primary healthcare provider (PHCP). If this was to occur, the nurse has one hour to inform the provider and obtain an order. Restraints are never as needed (PRN). They should be discontinued at the earliest possible time. When restraining a client, the reason for the restraint must be explained to the client and the behavior the client needs to demonstrate for the restraints to be discontinued. The nurse should observe the client at frequent intervals to offer nutrition & toileting, assess their behavioral status, obtain vital signs, and provide range of motion. These intervals are determined by the facility and the type of restraint—the more restrictive the restraint and the younger the client, the more frequent assessment. Restraints must be able to quickly be removed via a quick release buckle (knots are no longer recommended). The nurses' documentation must be comprehensive, describing the reasoning for the restraints, alternatives utilized, the education provided to the client, the type of restraint utilized, how it was secured, and the ongoing behavior necessary to continue the restraint. The nurse should also document the intervals at which the restraints were released. Last Updated - 01, Jun 2022

A pediatric client with pulmonary tuberculosis (TB) is scheduled to be admitted to the pediatric unit when the charge nurse learns the remaining private room on the unit was filled on the prior shift. No other pediatric TB clients are currently admitted. What is the most appropriate action for the charge nurse? A. Contact the infection control nurse B. Room the client with an uninfected client 6 feet apart C. Place the client with the varicella client currently in the airborne isolation room 6 feet apart D. Refuse to admit the pediatric TB client

Explanation Choice A is correct. The charge nurse should consult the infection control nurse for patient placement alternatives, as the pediatric client with pulmonary tuberculosis requires airborne isolation. Choice B is incorrect. The disease is transmittable through airborne droplets. If these two pediatric clients are placed in the same room, the uninfected child could acquire the TB infection through airborne droplets. Choice C is incorrect. Despite being in an airborne isolation room with one another, the TB pediatric client and the pediatric client positive for varicella are a risk to one another. These two clients should not be placed in the same room, as both illnesses are transmitted similarly, putting both clients at risk. Choice D is incorrect. Before refusing the admission, the charge nurse should consult the infection control nurse to discuss available options. If the issue remains unresolved, the charge nurse should also escalate the issue in an attempt to rectify the matter before refusing the pediatric client's admission. Learning Objective Recognize the need to contact the infectious disease nurse when faced with a patient placement matter involving an infectious disease matter. Additional Info Tuberculosis is a chronic, progressive mycobacterial infection, often with an asymptomatic latent period following the initial infection. Tuberculosis most commonly affects the lungs. Symptoms include productive cough, fever, weight loss, and malaise. Treatment is with multiple antimicrobial drugs (i.e., isoniazid, rifampin, pyrazinamide, and ethambutol) given concurrently. In airborne infection isolation (AII), the client is placed in an AII room. The main characteristics of AII rooms are that they have negative air pressure relative to the hall and 12 or more air exchanges per hour, of which at least two are outside air (for some older structures, six or more per hour are acceptable). Last Updated - 18, Oct 2022

The nurse is developing a health promotion teaching plan for a community group of middle-aged adults. Information about which immunizations should be included? A. Pneumococcal and meningococcal B. Pertussis, influenza, and meningococcal C. Influenza and pneumococcal D. Meningococcal and pertussis Submit Answer

Explanation Choice C is correct. Adults aged 50 years or older tend to have multiple chronic illnesses, in addition to an aging immune system. For this reason, the influenza vaccine is highly recommended. Choice A is incorrect. The pneumococcal vaccine protects against the most common pathogens that cause pneumonia. It is given every ten years. Choice B is incorrect. Immunization for pertussis is only indicated in children. Choice D is incorrect. Meningococcal vaccine is appropriate for adolescents and young adults living in congregate housing. NCSBN Client Need Topic: Physiological Integrity, Subtopic: Physiological Adaptation Last Updated - 08, Jan 2022

The nurse is discussing infection control practices in the nursing unit. Which client requires droplet precautions? A client with Select all that apply. A. diagnosed with rubella. B. a new diagnosis of diphthieria. C. receiving chemotherapy via an implanted port. D. pulmonary tuberculosis receiving nebulizer treatments. E. a skin abscess that tested positive for Klebsiella. F. Haemophilus influenzae, type b Submit Answer

Explanation Choices A, B, and F are correct. Diphtheria, rubella, and, Haemophilus influenzae, type b, all require droplet precautions. The personal protective equipment (PPE) necessary for droplet precautions is a surgical mask and gown. Choices C and D are incorrect. Airborne precautions are used for patients with infections spread through the air with small particles, such as pulmonary tuberculosis. A client receiving chemotherapy via an implanted port requires standard precautions unless the client has neutropenia. A skin abscess with Klebsiella warrants contact precautions. Learning Objective Recognize the diseases needing droplet isolation precautions.

The nurse is teaching a group of students on incident reports. Which of the following situations would require an incident report? Select all that apply. A client A. requesting to view their medical record. B. complaining about poor care from a nurse. C. leaving against medical advice (AMA). D. requesting an increase in pain medication. E. threatening a nurse with bodily harm. Submit Answer

Explanation Choices B, C, and E are correct. Incident (sometimes termed occurrence or event) reporting is required when any activity deviates from the norm. Events such as client complaints regarding their care, leaving against medical advice (AMA), and threatening a nurse with bodily harm are all examples of incidents requiring factual reporting. Choices A and D are incorrect. Incident reporting would be inappropriate for clients requesting to view their medical records. This is a right afforded to them. A request for an increase in pain medication or even a request for pain medication does not require reporting. Additional Info Incident (sometimes termed occurrence or event) reporting is a tool to mitigate future risks. Incident reporting should also be completed for events involving clients and visitors. Such events include: Verbal and physical displays of aggression Tampering with medical devices Reports of sexual or physical abuse by staff Adverse reaction to a blood transfusion Client elopement Damage (or loss) of client possessions Falls or injuries Injuries related to a medical device Complaints Medication and treatment errors Interfering with client care The incident should not be logged in the medical record or nursing notes. The documentation should be objective and factual and include what occurred, any injuries, if the provider was notified, care administered after the event, and any witnesses. Last Updated - 17, Sep 2022

The nurse is admitting a client who has cryptococcosis pneumonia. Which of the following actions would be appropriate for the nurse to take? A. Ensure a hand sanitizing station is near the client's room. B. Wear a surgical mask when working within three feet of the client. C. Keep the door to the client's room always closed. D. Place the client in a private room with monitored negative airflow. Submit Answer

Explanation Choice A is correct. Cryptococcosis pneumonia is a fungal infection not transmitted from human to human. Rather, this infection is opportunistic for individuals who are significantly immunocompromised. Standard precautions are necessary, which involve appropriate hand hygiene. Choices B, C, and D are incorrect. Cryptococcosis is a fungal infection commonly found in the soil that infects immunocompromised individuals. This pathogen requires standard precautions, so wearing a surgical mask, placing the client in a private room, and keeping the door shut are not necessary. Additional Info Cryptococcus neoformans is a fungal infection that is pathogenic for immunocompromised individuals. This pathogen is not transmitted from person to person and may trigger either pneumonia or meningitis. Individuals with human immunodeficiency virus (HIV) or acquired immunodeficiency syndrome (AIDS) are at significant risk for this infection if their CD4 count is significantly depressed. Treatment includes antifungals such as fluconazole. Last Updated - 13, Sep 2022

The nurse is conducting a staff conference regarding standard precautions. It would be correct for the nurse to state that hand washing with soap and water is required when A. hands are visibly soiled. B. collecting vital signs (VS). C. performing range of motion exercises. D. inputting data into the eletronic medical record (EMR).

Explanation Choice A is correct. Hand hygiene is a crucial component of standard precautions. It is the most important measure to prevent the spread of infections among clients. Usually, hand hygiene may be performed either by washing hands with soap and water or by using an alcohol-based sanitizer. However, in certain situations, it is required that hand hygiene is performed only by washing hands with soap and water. One of these situations is when the hands are visibly soiled. Choices B, C, and D are incorrect - Alcohol-based sanitizers may be used when the hands are not visibly soiled. In situations such as collecting vital signs, performing range of motion exercises, and inputting data into the electronic medical record (EMR) it is absolutely reasonable to use santizers because the risk of coming into contact with blood or bodily fluids that may contaminate hands is quite low. Additional Info According to the Centers for Disease Control and Prevention: Unless hands are visibly soiled, an alcohol-based hand rub is preferred over soap and water in most clinical situations due to evidence of better compliance compared to soap and water. Hand rubs are generally less irritating to hands and, in the absence of a sink, are an effective method of cleaning hands. Last Updated - 13, Oct 2022

The nurse is reviewing the laboratory results of a patient scheduled for surgery. Which of the following should be reported to the primary health care provider (PHCP)? A. Glycosylated hemoglobin (HbA1c) of 7.2% B. International Normalized Ratio (INR) of 3.5 C. Hematocrit (Hct) of 42% D. Blood urea nitrogen (BUN) level of 5 Submit Answer

Explanation Choice B is correct. An INR of 3.5 seconds is elevated and needs to be reported because the client may bleed. Choice A is incorrect. The HbA1c is elevated but would not impact a client scheduled for surgery. Choice C is incorrect. The hematocrit of 42% is within normal limits. Choice D is incorrect. A BUN level of 5 is decreased but poses no threat to the client. Last Updated - 31, May 2022

The nurse is discussing infection control guidelines with a coworker. It would be correct for the nurse to state that A. "The nurse should wear a surgical mask when transporting a client with active pulmonary tuberculosis (TB)." B. "Disposable utensils must be provided for a client infected with hepatitis B." C. "A surgical mask should be worn when working within three feet of the client infected with Neisseria meningitidis." D. "A surgical gown should be applied when entering a client's room with bacterial pneumonia." Submit Answer

Explanation Choice C is correct. Neisseria meningitidis is spread by infected droplets, and the nurse should wear a surgical mask while working within three feet of the client. Three feet is the distance for droplets to spread to another individual. If the client with Neisseria meningitidis should leave the room, they should wear a surgical mask. Choices A, B, and D are incorrect. When transporting a client with pulmonary tuberculosis, the client wears the surgical mask - not the nurse. Disposable dishes are not necessary to prevent the transmission of an infectious organism. The Centers for Disease Control and Prevention does not require disposable dishes for transmission-based precautions (contact, droplet, airborne). Bacterial pneumonia calls for droplet precautions (surgical mask), not a surgical gown, as that would be required for contact precautions.

The occupational health nurse is conducting an in-service on reducing back injuries. Which of the following statements, if made by a participant, would indicate a correct understanding of the conference? A. "I should keep my legs straight while lifting." B. "Heavy objects should be held away from my body." C. "I shouldn't twist while lifting an object." D. "I should keep a narrow base of support." Submit Answer

Explanation Choice C is correct. This is a true statement. Twisting while lifting an object will increase the likelihood of injury because twisting alters the individual's balance. Choices A, B, and D are incorrect. These statements are incorrect and require follow-up. While lifting, the legs should be appropriately bent, and the back should be straight. Heavy objects should be held near the body to maintain control and balance. A wide support base (feet should be shoulder width apart) is necessary to maintain balance. Additional Info Effective Measures to Prevent Back Injury Include Have the necessary assistance to move the object. Planning the move and communicating with the other individual who will assist you. Using the shoulder, upper arms, hips, and thighs as the predominant muscles to help with the move. Keep objects close to your body when lifting or carrying objects. Avoid twisting by using your feet to turn your body. Use a mechanical lift when necessary. Last Updated - 21, Aug 2022

The nurse is caring for a client in bilateral soft wrist restraints. The nurse should assess the client's Select all that apply. A. behavioral status. B. skin integrity. C. bowel sounds. D. neurovascular status. E. need for the restraint.

Explanation Choices A, B, D, and E are correct. Assessing a client in soft wrist restraints should occur at a minimum of every two hours (or per facility policy). Pertinent assessments should include the client's behavioral status, skin integrity, neurovascular status (pulses, capillary refill), and the continued need for the restraint. Choice C is incorrect. Assessing the client's bowel sounds is not pertinent related to restraints. This would be relevant if the client had a nasogastric tube (NGT) or an acute gastrointestinal problem. Additional Info Restraints should be used as a last resort if alternative methods are not effective. A nurse should never threaten a client with restraints. This is considered assault. The nurse may place a client who is violent in restraints without an order from the primary healthcare provider (PHCP). If this was to occur, the nurse has one hour to inform the provider and obtain an order. Restraints are never as needed (PRN). They should be discontinued at the earliest possible time. When restraining a client, the reason for the restraint must be explained to the client and the behavior the client needs to demonstrate for the restraints to be discontinued. The nurse should observe the client at frequent intervals to offer nutrition & toileting, assess their behavioral status, obtain vital signs, and provide range of motion. These intervals are determined by the facility and the type of restraint—the more restrictive the restraint and the younger the client, the more frequent assessment. Restraints must be able to quickly be removed via a quick release buckle (knots are no longer recommended). The nurses' documentation must be comprehensive, describing the reasoning for the restraints, alternatives utilized, the education provided to the client, the type of restraint utilized, how it was secured, and the ongoing behavior necessary to continue the restraint. The nurse should also document the intervals at which the restraints were released. Last Updated - 01, Jun 2022

You are providing education on home safety to a group of new parents. Which of the following educational points are important to include? Select all that apply. A. Use stair gates to keep children off the stairs until they are in kindergarten. B. Cover the electrical outlets by the time your infant is 7 months old. C. Place locks on toilet lids by the time your infant is 7 months old. D. Move items on coffee tables to areas that cannot be reached before your child is a year old. Submit Answer

Explanation Choices B and C are correct. It is important to teach parents that electrical outlets should have plug covers in place by the time their child is 7-months-old. At this age, the infant will be able to crawl and will be reaching out to touch unfamiliar things. This is when electrical outlets start to pose a risk and should therefore be covered (Choice B). It is important to teach parents that toilet lids should have locks on them by the time their child is 7 months old. At this age, the infant will be able to crawl and will be reaching out and pulling themselves up on things. This is when toilets start to pose a risk and should have locks placed on their lids so that they cannot fall into them (Choice C). Choice A is incorrect. It is not necessary to use stair gates until the child is in kindergarten, as they should be able to safely navigate stairs on their own by 3 to 4 years of age. Advise parents to use stair gates until their child is 3 to 4 years old and is able to walk up and down the stairs without their support. Choice D is incorrect. This should be done by the time the child is 7 months old. A child who is developing normally will be crawling, reaching, and pulling themselves up on things by the time they are 7 months old, so this is when items on tables should be moved for their safety. NCSBN Client Need: Topic: Effective, safe care environment, Subtopic: Coordinated care; Pediatric Development Last Updated - 26, Jan 2022

The nurse is instructing a nursing student on the correct application of personal protective equipment (PPE). The nurse should tell the student to apply the PPE in what order? Place each action in the correct order. Secure the mask Don gloves Apply the goggles/face shield Apply the gown Submit Answer

Explanation For donning personal protective equipment, the nurse should instruct the student to Apply the gown that should be securely fastened behind the neck and waist. Secure the mask that should extend below and under the chin. Fit the goggles/face shield. Don gloves that are a snug fit. Additional Info For donning personal protective equipment, the nurse should instruct the student to Apply the gown that should be securely fastened behind the neck and waist. Secure the mask that should extend below and under the chin. Fit the goggles/face shield. Don gloves that are a snug fit. For doffing personal protective equipment, the nurse should instruct the student to Gloves Goggles/face shield Gown Mask Last Updated - 29, Apr 2022

The spouse of your elderly male client tells you that her husband becomes so cold when he is outdoors that she warms up his car in the garage before helping him get into his car. How should you respond to her statement? A. "You should not warm up the car in the garage because it is hazardous." B. "That is a good idea, as your husband frequently complains about being cold." C. "You can also dress him in warmer clothing than needed so he is not cold." D. "That is the most foolish thing I have heard in a long time. You have to stop that." Submit Answer

Explanation Choice A is correct. After informing the client's wife that she " . . . should not warm up the car in the garage because it is hazardous," you would explain how a buildup of carbon monoxide would occur and why it be deadly. Based on her statement, the client's spouse demonstrated a knowledge deficit by telling you that she warms up the car in the garage. In response, you must address her knowledge deficit with patient (or caregiver) education. Choice B is incorrect. This is an incorrect statement that reinforces a dangerous behavior that places the client at risk for carbon monoxide poisoning. Choice C is incorrect. Suggesting to the client's spouse that the client should be dressed in warmer clothing does not resolve the issue regarding the risk of carbon monoxide poisoning. Choice D is incorrect. A statement made in this manner is not therapeutic communication, as it does not address the spouse's learning deficit regarding the potential for carbon monoxide poisoning. If a statement is made to a client or a caregiver similar to this, the nurse will likely alienate the receiver of the message from that point forward. Learning Objective Identify a therapeutic response that educates a client and his caregiver on the dangers of carbon monoxide poisoning. Additional Info Carbon monoxide poisoning causes acute symptoms such as headache, nausea, weakness, angina, dyspnea, loss of consciousness, seizures, and coma. Weeks later, neuropsychiatric symptoms may develop. Diagnosis is made by carboxyhemoglobin levels and arterial blood gases (ABGs), including measured oxygen saturation. Treatment is with supplemental oxygen. Household carbon monoxide detectors may aid in prevention. Last Updated - 30, Aug 2022

The nurse is caring for a client who has rubella. The nurse should isolate the client using which of the following? A. Airborne precautions B. Droplet precautions C. Contact precautions D. Standard precautions Submit Answer

Explanation Choice B is correct. Rubella is known as German measles and requires droplet precautions. The nurse is right to wear a surgical mask when engaging with the client. The transmission mode for rubella is a droplet mode of communication where the spread occurs with particle drops larger than 5 microns. Choices A, C, and D are incorrect. Do not confuse rubeola with rubella. Rubeola (measles) requires airborne precautions, whereas rubella needs droplet precautions. Airborne transmission occurs when the pathogen is carried in dust or droplets in the air that remains in place for sufficient time to infect a person exposed to this air. Contact transmission precautions are used for infectious diseases spread by contact with the client or the client's environment. Contact transmission precautions prevent infections such as C. difficile and shigella, but not rubella. Standard precautions are a set of infection control practices used to prevent the transmission of diseases that can be acquired by contact with blood, body fluids, non-intact skin (including rashes), and mucous membranes. Droplet precautions involve wearing a surgical mask and are used for rubella. Standard precautions do not include routinely wearing a surgical mask and are insufficient to prevent rubella transmission. Additional Info Personal Protective Equipment (PPE) required for a client with droplet precautions is a surgical mask. The risk of transmitting a droplet pathogen is likely when the nurse is within three feet of the individual infected. Conditions requiring droplet precautions include: Rubella Influenza Pertussis Bacterial meningitis Pneumonic plague Diphtheria (Pharyngeal) Mumps Rhinovirus

The nurse is visiting an older adult client with impaired vision. It would be necessary for the nurse to follow up if the client states which of the following? Select all that apply. A. "I secured my throw rugs to the floor with tape." B. "I switched to using an electric shaver instead of a razor." C. "I usually sit in a recliner while I listen to the television." D. "I use different shaped containers with lids to organize my medications." E. "I use the upstairs bathroom instead of the one downstairs." Submit Answer

Explanation Choices A and E are correct. An older patient with impaired vision that lives alone has significant risk factors for falls. The nurse should follow up if the client states that they secured the scattered rugs with tape. The client should not have any scattered rugs. Finally, a client climbing the stairs to use the bathroom increases the risk of falls. The nurse should advise the patient to use the closest bathroom. Choices B, C, and D are incorrect. It is appropriate for a client to use an electric shaver versus manual shaving because of the lessened risk of injury. Reclining while watching television poses no threat to the client, and the client should be encouraged to use different shaped containers to organize their medications. Additional Info The nurse must recognize fall risk factors, including older age, sensory impairments, ambulation device(s), incontinence, and certain medications (antihypertensives, benzodiazepines, opioids). The nurse must work to reduce the client's risk of injury if these risk factors are present. Last Updated - 13, Jan 2022

A client requests to change rooms after overhearing that their roommate is positive for the human immunodeficiency virus (HIV). The nurse should take which appropriate action? A. Relocate the client to a private room B. Ask the client to elaborate on their concern C. Notify the risk manager of the request D. Place an additional divider in-between the two beds

Explanation Choice B is correct. Asking the client to elaborate on their concern is the most logical and therapeutic action. The client is likely misinformed about the disease transmission of HIV, and the nurse should encourage the client to verbalize their concerns. It also is appropriate for the nurse to respond to any misconceptions the client may have with compassion and facts. Choices A, C, and D are incorrect. These actions are incorrect. Based on the information provided, the client should not be relocated to a private room. Additionally, if the client were to be relocated, a private room would not be necessary, as another semi-Private room would be appropriate. The risk manager does not need to be notified of the request. This request would be irrelevant to a risk manager. Finally, an additional divider is unnecessary because this would further fuel the hysteria surrounding HIV. A client with HIV can be roomed with another individual, and standard precautions should be utilized. Additional Info The worst symptom of HIV is the stigma. Many misconceptions exist regarding the transmission of HIV. Image Credit: CDC.gov Last Updated - 23, Sep 2022

Which of the following vaccines contains a live virus? Correct A. IPV B. DTaP C. Varicella D. Hepatitis B

Explanation Choice C is correct. Varicella is a live virus. Currently, the available live attenuated viral vaccines are measles, mumps, rubella, vaccinia, varicella, zoster (which contains the same virus as varicella vaccine but in a much higher amount), yellow fever, rotavirus, and influenza (intranasal). Choice A is incorrect. IPV is an inactivated polio vaccine. Choice B is incorrect. DTaP contains inactivated forms of the toxin produced by the bacteria that cause the three diseases Diphtheria, Tetanus, and Pertussis. Choice D is incorrect. Hepatitis B vaccine is a genetically engineered (human-made in the laboratory) piece of the virus. It does not contain a live virus. NCSBN Client Need, Topic: Health Promotion and Maintenance, Subtopic: Immunizations Last Updated - 03, Dec 2021

Which of the following children would the nurse identify as a priority for having the greatest risk for choking and suffocating? A. A toddler playing with his 9-year-old brother's construction set. B. A 5-year-old eating yogurt for a snack. C. An infant asleep in her crib without a blanket. D. A 3-year-old drinking a glass of juice. Submit Answer

Explanation Choice A is correct. A young child may place small or loose parts of toys in his mouth. A toy that is safe for a 10-year-old child could be deadly for a toddler. Choice B is incorrect. 5-year-old eating yogurt is not a safety concern. Choice C is incorrect. An infant sleeping in an empty crib is not a safety concern. According to the American Academy of Pediatrics, blankets and pillows increase the risk of suffocation and Sudden Infant Death Syndrome (SIDS). Several infants die each year while sleeping, and the cause is attributed to SIDS, suffocation, entrapment, or strangulation. Blankets increase the risk of all these four reasons. Choice D is incorrect. A 3-year-old drinking a glass of juice is not a safety concern. NCSBN Client Need Topic: Safe and Effective Care Environment, Subtopic: Safety and Infection Control - Choking Hazards Last Updated - 15, Feb 2022

The nurse is caring for a client who has a prescribed regular insulin sliding scale. At 0800, the client's capillary blood glucose (CBG) was 258 mg/dl. At 1215 the CBG was 288 mg/dl. At 1730 the CBG was 254 mg/dl. The nurse should do which of the following at 1730? See the table below. Select all that apply. A. Administer 8 units of regular insulin B. Administer 6 units of regular insulin C. Notify the primary health care provider (PHCP) D. Withhold the prescribed insulin E. Modify the client's prescribed diet to low sodium Submit Answer

Explanation Choices B and C are correct. The client's blood glucose has been above 250 mg/dL for three consecutive readings and the physician needs to be notified. In addition, the sliding scale prescribes six units of insulin based on the 17:30 CBG result. Choices A, D, and E are incorrect. These are inappropriate. Modifying the diet to low sodium requires a prescription from the physician and would be ineffective for a client who is hyperglycemic. Last Updated - 10, Feb 2022

A client with intermittent insomnia asks the nurse about herbal supplements to improve sleep. Which product should the nurse recommend the patient ask her care provider about? A. Raspberry leaf tea B. Valerian root C. Glucosamine D. Black Cohosh Submit Answer

Explanation Choice B is correct. Valerian root is a common herbal remedy used to treat occasional insomnia but may interact with some medications. Patients should be encouraged to discuss herbal remedies with their doctors. Choice A is incorrect. Raspberry tea leaf is a popular herbal tea used to induce labor and expedite the shrinking of the uterus post-partum. It is not used to treat insomnia. Choice C is incorrect. Glucosamine is a commonly recommended amino sugar used to promote joint health. It does not correct insomnia. Choice D is incorrect. Black cohosh is an herbal supplement sometimes used to induce abortion in the early first trimester or to induce labor after a woman is forty weeks pregnant. It does not help with insomnia. NCSBN client need Topic: Psychosocial Integrity / Cultural Awareness Last Updated - 12, Feb 2022

What consideration should the nurse keep in mind regarding the use of side rails for a confused patient? A. They prevent confused patients from wandering. B. A history of a previous fall from a bed with raised side rails is insignificant. C. A person of small stature is at increased risk for injury from entrapment. D. Alternative measures are ineffective to prevent wandering.

Explanation Choice C is correct. Studies of restraint-related deaths have shown that people of small stature are more likely to slip through or between the side rails. Choice A is incorrect. The desire to prevent a patient from wandering is not sufficient reason for the use of side rails. Choice B is incorrect. A history of falls from a bed with raised side rails carries a significant risk for a future serious incident. Choice D is incorrect. Creative use of alternative measures indicates respect for the patient's dignity and may, in fact, prevent more serious fall-related injuries. NCSBN Client Need Topic: Safe and Effective Care Environment, Subtopic: Safety and Infection Control; Using Side Rails as Restraints Last Updated - 23, Jan 2022

When doing presurgical assessments of patients in an ambulatory care center, which patient would the nurse report to the surgeon as possibly needing surgery to be postponed? A. A 20-year-old patient who is a vegan. B. An elderly client who takes daily nutritional drinks. C. A 40-year-old patient who takes ginkgo biloba and an aspirin daily. D. An infant who is breastfeeding.

Explanation Choice C is correct. The patient taking Ginkgo Biloba (herbal), aspirin, and vitamin E may have to postpone surgery due to the increased risk for excessive bleeding, as each of these substances has antiplatelet and/or anticoagulant properties. Ginkgo Biloba can further potentiate aspirin's antiplatelet effect and may increase the bleeding tendency. Surgeons usually require aspirin and other antiplatelet agents to be held at least seven days before significant surgeries. Choices A, B, and D are incorrect. Being a vegan, drinking nutritional drinks, and breastfeeding do not adversely affect surgery outcomes. Learning Objective Recognize that Ginkgo Biloba can potentiate aspirin's antiplatelet effects and may increase the bleeding time. Additional Info Patients commonly use certain herbal supplements. Nurses should be aware of the side effects of these herbal supplements and their interactions with other commonly prescribed medications.

The nurse assesses a client's central venous catheter dressing, and it appears loose and wet. The nurse should take which action? A. Reinforce the dressing with paper tape B. Remove the dressing and the central vascular device C. Apply a clean occlusive dressing to the site D. Clean the site and apply a new sterile dressing Submit Answer

Explanation Choice D is correct. When moisture comes into contact with a sterile dressing, this causes contamination. The nurse should prepare to change the dressing using medical asepsis to remove the old dressing and surgical asepsis to apply the new dressing. Choices A, B, and C are incorrect. Reinforcing the dressing with paper tape would be inappropriate. Moisture causes contamination, and the nurse should intervene and change the dressing. Removing the central line would be inappropriate as the issue is with the dressing - not the catheter. An occlusive dressing does not need to be applied as the old dressing needs to be removed, the site cleaned with chlorhexidine, and a new central line dressing applied. Additional Info The central line should be anchored with a securement device and covers the site with a sterile bio-occlusive dressing. Central line dressings should be changed at least every 7 days or immediately if dressing integrity is disrupted (e.g., lifted/detached on any border edge or within transparent portion of dressing, visibly soiled, presence of moisture, drainage, or blood). Last Updated - 02, Jun 2022

You are working in the emergency department when a 23-year-old woman comes in after being bitten by multiple fire ants. You examine her and notice that there are many fluid-filled bites on her legs and ankles. She is complaining of numbness in her face and you notice swelling around her lips. She complains of shortness of breath and her respiratory rate is 28 breaths per minute. You hear wheezing when you auscultate her lungs. Her heart rate is 110/minute and her blood pressure is 82/40 mmHg. You have the following orders for this patient: Administer epinephrine Ensure a patent airway Administer an antihistamine Place on a cardiac monitor to analyze the heart rhythm As you work to prioritize these tasks, you know that the best sequence for doing them is: Administer epinephrine Administer an antihistamine Ensure a patent airway Place on a cardiac monitor to analyze the heart rhythm Submit Answer

Explanation Correct ordered sequence: The nurse should recognize that this patient is in anaphylaxis, which is a life-threatening allergic reaction to ant bites. Although all of these tasks should be done for this patient, the one with the highest priority is to ensure that the woman has a patent airway. If an advanced airway is required, the nurse should call for assistance to complete this task. In most patients, positioning the patient and providing supplemental oxygen will be sufficient. This should be followed very quickly by the administration of epinephrine IM or SC. Epinephrine is the drug of choice for anaphylaxis since it helps to increase blood pressure and inhibits the release of mediators, causing the reaction. Next, the nurse should administer an antihistamine such as diphenhydramine. This should never replace the administration of epinephrine because the onset of action of antihistamines is slower than epinephrine. Often, clinicians will administer both diphenhydramine and ranitidine since one is an H1 blocker, and the other is an H2 blocker. Finally, it is essential to connect the patient to a cardiac monitor since anaphylaxis can lead to cardiac arrhythmias. If other team members are present, this can often be accomplished simultaneously with the other tasks, but connecting the patient to a cardiac monitor is a lower priority than the other actions. NCSBN Client Need Topic: Management of Care, Sub-Topic: Establishing Priorities, Critical Care Last Updated - 24, Jan 2020

A 60-year old adult walks into the clinic with a forehead laceration. He was going 45 mph on his motorcycle when he skidded on the gravel and fell off his bike. He isn't sure if he hit his head. After checking for significant bleeding and apparent signs of injury, what is the first intervention the nurse should do? A. Place a c-collar on the patient's neck. B. Take the patient back to a procedure room to stitch the laceration. C. Palpate the patient's abdomen to check for internal bleeding. D. Check the patient's pupils. Submit Answer

Explanation Choice A is correct. After checking for apparent signs of illness, this patient needs c-spine precautions placed immediately. This patient was going faster than 30 mph on a motorcycle. We are not sure if he was wearing a helmet, but with the mechanism of injury, he could have a possible c-spine injury. This is important in preventing any spinal cord damage from occurring. Choice B is incorrect. Even though this intervention needs to be performed at some point, it is not the most important thing to do. A laceration may need pressure to stop the bleeding, but it can wait to be sutured. Choice C is incorrect. This step comes later in the head-to-toe assessment. The emergency physician will utilize an ultrasound machine to assess for any internal bleeding or complications. Choice D is incorrect. This step comes later in the head-to-toe assessment. This will happen after airway/c-spine, breathing, and circulation are assessed. The pupils should be assessed in this situation because the patient isn't sure if he hit his head on the ground. This could result in possible brain injury, including illness. NCSBN Client Need Topic: Physiological Adaptation, Sub-topic: Alterations in Body Systems Last Updated - 01, Feb 2022

The nurse is caring for a client who has pertussis. Which infection control precaution should the nurse implement? Select all that apply. A. Wear a surgical mask prior to entry B. Provide disposable dishes for meals C. Keep the patient's room door closed D. Provide the patient with a portable fan E. Maintain negative air pressure F. Apply an N95 mask to the patient during transport G. Place the patient in a room near the nurse's station Submit Answer

Explanation Choice A is correct. Infected droplets spread pertussis. Therefore, the nurse must wear a surgical mask when providing care. Visitors must also wear the surgical mask, and during client transport, the patient should wear a surgical mask - not N95. Choices B, C, D, E, F, and G are incorrect. Disposable dishes and utensils are not necessary for any isolation precaution. Further, this client requires droplet precautions, and the door may remain open with no need for negative pressure. The nurse should not provide the client with a fan as the fan will spread the infected droplets around the room. The client doesn't need to be placed near the nurse's station as this is an intervention for a client at risk for falls. Please note, that select all that apply items may require one or all responses according to the NCSBN (the authors of the NCLEX) Additional Info Transmission-based precautions for pertussis include droplet precautions such as wearing a face mask and appropriate hand hygiene. The nurse may only room a patient with pertussis with another client with the same infection. Keeping the client's room door closed and maintaining negative air pressure are appropriate interventions for airborne isolation (unlike in this patient, with respiratory droplet precautions).

Which action taken by the school nurse will have the most impact on the incidence of infectious disease in the school? A. Ensure that students are immunized according to national guidelines. B. Provide written information about infection control to all patients. C. Make soap and water readily available in the classrooms. D. Teach students how to cover their mouths when coughing. Submit Answer

Explanation Choice A is correct. The incidence of once-common infectious diseases such as measles, chickenpox, and mumps have been effectively reduced by immunization of all school-aged children. School-aged children are at risk for problems such as exposure to viruses, respiratory infections, and parasitic infections (such as scabies or lice). Vaccination protects children from serious illness and complications of vaccine-preventable diseases which can include amputation of an arm or leg, paralysis of limbs, hearing loss, convulsions, brain damage, and death. Choices B, C, and D are incorrect. While these are helpful, receiving proper and timely immunizations has a great impact. NCSBN Client Need Topic: Physiological Integrity, Subtopic: Reduction of Risk Potential Last Updated - 27, Aug 2021

The nurse is caring for a group of premature infants. Which action is most important in preventing healthcare-acquired infection? A. Performing frequent hand hygiene B. Disinfecting commonly touched surfaces C. Screening visitors for illness D. Administer prophylactic antibiotics Submit Answer

Explanation Choice A is correct. The most important action a nurse can take to prevent a healthcare acquired infection is to frequently wash their hands. Hand hygiene is a proven and effective measure to decrease the transmission of pathogens. The nurse should wash their hands when they are visibly soiled, before and after contact with the client. Alternatively, the nurse may use alcohol-based sanitizers. Choices B, C, and D are incorrect. These measures are useful in preventing disease transmission however, handwashing is critical to reducing the transmission of microorganisms. Prophylactic antibiotic use is often discouraged because of the exposure to adverse effects such as gastrointestinal illness and antibiotic resistance. Additional Info Good hand hygiene is a pillar in healthcare. As a key reminder, hands should be rinsed by keeping hands down and elbows up. Finally, hands should be dried thoroughly from fingers to wrists with a paper towel or single-use cloth. Last Updated - 10, Sep 2022

A nurse is preparing a client's intravenous (IV) infusion. As the nurse was preparing to attach the distal end of the IV tubing to the client's needleless access device, the exposed tubing slipped and hit the top of the client's bedside table. Which of the following is the most appropriate action by the nurse? A. Replace the IV tubing with new tubing B. Discard the client's current needleless access device and replace it with a new one C. Wipe the distal end of the tubing with povidone-iodine to render it sterile D. Clean the needleless access device with an alcohol swab

Explanation Choice A is correct. The nurse should replace the IV tubing as the existing tubing has now been contaminated and places the client at increased risk for systemic infection due to direct infusion into the bloodstream. Choice B is incorrect. There is no need to discard and replace the client's needleless access device, as this is not the device that has been contaminated. Choice C is incorrect. Wiping the contaminated end of the tubing with povidone-iodine is insufficient and will not render this tubing sterile. Choice D is incorrect. Although the client's needleless access device will be cleaned with an alcohol swab or other aseptic before connecting the intravenous infusion (after new tubing has been attached), here, for purposes of this question, there is no need to clean the client's needleless access device with an alcohol swab for purposes of decontamination. Following the exposed tubing slipping and hitting the top of the client's bedside table, the client's needleless access device was not the item that was contaminated. Learning Objective Recognize that a breach in infection control practices via contamination of intravenous tubing will result in the need to replace the tubing prior to use. Additional Info The registered nurse is ultimately accountable for all aspects of infusion therapy and delegation of associated tasks. There is no specific timeframe that short peripheral catheters may remain in place. To reduce infection risk in needleless systems, implement the following interventions: Clean all needleless system connections vigorously with an antimicrobial (usually 70% alcohol or alcohol and 2% chlorhexidine swabs) for 10 to 15 seconds before connecting infusion sets or syringes. Do not tape connections between tubing sets. Use evidence-based hand hygiene guidelines from the CDC and OSHA. Last Updated - 25, Sep 2022

The nurse is observing a student prepare to perform a sterile procedure. Which action by the student would require follow-up? The student A. reaches over the sterile field to grab sterile gloves. B. establishes the sterile field on a dry surface. C. uses slow movements when setting up sterile drapes. D. keeps the sterile field at their waist level. Submit Answer

Explanation Choice A is correct. This action requires follow-up because reaching over the sterile field may cause contamination. Choices B, C, and D are incorrect. These actions reflect appropriate sterile technique. Sterile fields should be established on dry surfaces because moisture attracts microorganisms. Using slow movements is an effective measure when setting up and performing a sterile procedure. Using slow movements will decrease stirring up dust, lint, and infectious microorganisms, which may cause contamination. The sterile field should be at waist level or above. An object held below the waist or above the chest is considered contaminated. Additional Info General guidelines when it comes to sterile technique: A dry surface is needed for a sterile field. A sterile object placed below the waist is considered contaminated. A 2.5-cm (1-inch) border around any sterile drape/wrap is considered contaminated and can be touched with clean fingers. Never reach over a sterile field. The sterile object remains sterile only when touched by another sterile object. Use slow movements when setting up sterile drapes. Last Updated - 13, Sep 2022

What is the most important factor to consider when assessing a home health patient on the risk of falls? A. Correct illumination of the environment B. Amount of regular exercise C. The resting pulse rate D. Status of salt intake

Explanation Choice A is correct. To prevent falls; the environment should be well lit. Night lights should be used if necessary. Other factors in assessing include removing loose scatter rugs, removing spills, and installing handrails and grab bars as appropriate. While home health nurses cannot expect to change a family's living space and lifestyle, they can express their concern and react appropriately when a situation suggests that an injury is imminent. Nurses must document information they provide and the family's response to the instruction and make ongoing assessments about the family's use of safety precautions. Walkways and stairways (inside and outside) should also be inspected. Note any uneven sidewalks or paths, broken or loose steps, absence of handrails, or placement on only one side of stairs, insecure bars, congested hallways, or other traffic areas, and adequacy of lighting at night. Some important things for the home health nurse to assess, educate the patient about, and document include: Floors: Note uneven and highly polished or slippery floors and any unanchored rugs or mats. Furniture: Note the hazardous placement of furniture with sharp corners. Note chairs or stools that are too low to get into and out of or that provide inadequate support. Bathroom(s): Note presence of grab bars around tubs and toilets, nonslip surfaces in bathtubs and shower stalls, handheld showerhead, adequacy of night lighting, need for a raised toilet seat or bath chair in tub or shower, ease of access to shelves, and water temperature regulated at a maximum of 49 °C (120 °F). Choice B is incorrect. The amount of regular exercise is not the most critical factor to assess. It is only indirectly related. Choice C is incorrect. The resting pulse rate is not related to preventing falls. Choice D is incorrect. The salt intake is not directly related to preventing falls. NCSBN Client Need Topic: Safe and Effective Care Environment, Subtopic: Safety and Infection Control, Selected Dimensions of Home Health Nursing Last Updated - 31, Jan 2022

The nurse is observing the surgical aseptic technique of a nursing student. Which observation by the nurse requires follow-up? A. Spills sterile water onto the sterile field. B. Holds their hands above their elbows during handwashing. C. Has sterile gauze placed into the sterile field D. Keeps the sterile field above their waist. Submit Answer

Explanation Choice A is correct. When a sterile surface comes in contact with a liquid, the sterile object or field becomes contaminated. The nurse should intervene because microorganisms travel to the sterile object if moisture leaks through the protective covering of a sterile package. Choices B, C, and D are incorrect. These observations all adhere to surgical asepsis. Fluid flows in the direction of gravity. A sterile object becomes contaminated if gravity causes a contaminated liquid to flow over the surface of the object. Thus, it is appropriate for handwashing to occur with their hands over their elbows. Sterile gauze may be placed on the sterile field as long as the opening of the package is not torn and the object touches the border of the sterile field. The sterile field should be above the waist. Additional Info Surgical asepsis prevents contamination of an open wound, serves to isolate an operative or procedural area from an unsterile environment, and maintains a sterile field for surgery or procedural intervention. Surgical asepsis differs from medical asepsis in that surgical asepsis aims to completely kill microorganisms, whereas medical asepsis prevents the microorganisms from spreading. Last Updated - 02, Jun 2022

Your client has a stat order for a cooling or hypothermia blanket. After you call the appropriate department, the cooling blanket is delivered to your nursing care unit. What is the first thing you should do concerning this stat order? A. Inspect and run the equipment prior to use. B. Immediately use the cooling blanket for the client because it is a stat order. C. Ask the engineering department to perform preventive maintenance on it. D. Inspect the blanket for any frayed cords before to protect against fire. Submit Answer

Explanation Choice A is correct. You must thoroughly inspect and run the equipment before use to ensure that it is appropriately functioning BEFORE it is used. This inspection should include an overall assessment for frayed electrical cords and documented evidence that the piece of equipment has had the mandated preventive maintenance and safety inspections according to the facility's policies and procedure. Choice B is incorrect. You would not immediately use the cooling blanket for the client just because it is a stat order because other preventive measures must be taken first before using it. Choice C is incorrect. You would not ask the engineering department to perform preventive maintenance because you should be able to see documented evidence that the preventive maintenance was done on the sticker that is affixed to the piece of equipment. Choice D is incorrect. You would not merely inspect the blanket for any frayed cords before use to protect against fire. Last Updated - 14, Aug 2021

The patient is experiencing post-operative tachycardia with low blood pressure. The nurse should be most concerned about which of the following surgical complications? A. The development of an infection B. Hemorrhage C. Wound dehiscence D. Hematoma

Explanation Choice B is correct. A patient with low blood pressure and tachycardia after a surgical procedure may be experiencing an illness. Blood loss results in lowered blood pressure and the heart rate increases to compensate. Choice A is incorrect. The development of an infection after surgery usually presents with tachycardia and fever. Hypotension may or may not be present. Choice C is incorrect. Wound dehiscence occurs when the edges of a surgical site rupture. It is not categorized by tachycardia and low blood pressure. Wound dehiscence is a serious complication of surgery and needs immediate treatment. Choice D is incorrect. A hematoma, or a bruise, is a non-serious occurrence after surgery. It is caused by a collection of blood beneath the skin's surface. NCSBN client need Topic: Physiological Integrity, Reduction of Risk Potential Last Updated - 13, Feb 2022

Which of the following findings would prompt immediate investigation when performing an assessment of a patient on a medical/surgical unit? A. Bowel sounds of 14 per minute B. High-pitched bowel sounds at a rate of 4 per minute C. Bowel sounds greater than 60 per minute D. Low-pitched bowel sounds at a rate of 30 per minute

Explanation Choice B is correct. Bowel sounds less than 5 per minute may indicate blockage and should be evaluated. Bowel sounds are high-pitched, occasional gurgles, or clicks that last from one to several seconds. They occur every 5 to 15 seconds in the average adult. Choice A is incorrect. Bowel sounds of 14 per minute are considered normal. Choice C is incorrect. Although bowel sounds that are more than 30 per minute are considered hyperactive, it is not as immediate a concern as choice B. Choice D is incorrect. Bowel sounds usually are high-pitched. However, the rate of bowel sounds is within normal limits. This answer choice does not pose as much concern as choice B. NCSBN Client Need Topic: Physiological Integrity, Subtopic: Reduction of Risk Potential - Auscultating the Abdomen Last Updated - 14, Feb 2022

The nurse is caring for a client who has pulmonary tuberculosis (TB). Which infection control measure should the nurse implement? A. Restrict visitors who are pregnant. B. Remove any portable fans in the room. C. Wear a dosimeter badge during patient care. D. Place the patient further away from the nursing station.

Explanation Choice B is correct. Fans should be removed from a room for a client with droplet or airborne precautions. Fans may propel the transmission of a pathogen. A client with pulmonary tuberculosis should be isolated using airborne precautions. Choices A, C, and D are incorrect. Restricting visitors who are pregnant is not an infection control measure necessary for pulmonary tuberculosis. It would also be inappropriate to wear a dosimeter badge or place the patient further away from the nursing station. Radiation exposure is not a risk associated with TB; thus, a dosimeter badge is unnecessary. Additional Info For a client with suspected or active pulmonary tuberculosis, the following infection control measures should be taken - Initiate and maintain airborne precautions. Keep the door closed to promote negative pressure. An appropriate respirator must be worn during direct care. Strict adherence to hand hygiene. If the client needs to leave the room, they should wear a surgical mask. Last Updated - 31, Dec 2021

The community health nurse is doing a home visit on a client that was admitted to the hospital two weeks ago for hypertension. The nurse notes that the client was prescribed amlodipine 5 mg daily and was advised to lose weight. The nurse should be concerned when the client notes which of the following during his visit? A. The patient states that he has already enrolled himself in a gym and is getting dietary counseling from a nutritionist. B. The nurse notes the patient drinking grapefruit juice. C. The patient asks the nurse multiple questions regarding how he can follow his treatment regimen. D. The patient stated that he has had an episode of dizziness a day after he was discharged but has since been fine. Submit Answer

Explanation Choice B is correct. Grapefruit juice and calcium channel blockers may combine to cause toxic effects. This should cause concern to the nurse and should necessitate further teaching regarding calcium channel blockers (ie, amlodipine). Choice A is incorrect. The patient stating that he is already starting to exercise and modify his diet is a positive sign that he is complying with his treatment. Choice C is incorrect. The patient asking various questions regarding his treatment signifies to the nurse that the patient is eager and willing to undergo therapy. The nurse should encourage the patient. Choice D is incorrect. The dizziness that the patient has stated is just a common side effect of his medication (calcium channel blocker). Last Updated - 01, Feb 2

The nurse is evaluating a patient with symptoms of metabolic acidosis. Which of the following is not a cause of metabolic acidosis? A. Severe diarrhea B. Hyperventilation C. Starvation D. Diabetes mellitus Submit Answer

Explanation Choice B is correct. Hyperventilation due to asthma, anxiety, or high altitude may lead to respiratory alkalosis. Unless it is quickly corrected, acidosis and alkalosis can have severe or fatal consequences. The nurse needs to understand possible causes and identify symptoms as soon as possible. Note: Acidosis and alkalosis are not diseases, but instead signs of an underlying disorder. The primary treatment of acid-base disorders is targeted at correcting the underlying cause. Choices A, C, and D are incorrect. These are all possible causes of metabolic acidosis. Therefore, not the answers to the question. NCSBN Client Need Topic: Physiological Integrity, Subtopic: Pharmacological Therapies Last Updated - 15, Feb 2022

The school nurse is talking to a group of mothers regarding poison prevention and management. Which statement by the mothers indicates a need for further teaching? A. "I need to properly label the containers of poisonous liquids." B. "I need to make my child vomit in the instance he ingests gasoline." C. "I can give my child milk or some water to dilute the poison while I rush him to the hospital." D. "All poisonous materials should be stored away from children." Submit Answer

Explanation Choice B is correct. Induction of vomiting when a victim has ingested hydrocarbons is contraindicated. Vomiting may lead to inhalation of the poison, worsening the situation. Choice A is incorrect. This is a correct statement. Proper labeling can help prevent accidental ingestion of poisons at home. Choice C is incorrect. This is a correct statement. Diluting the poison can buy some time in getting the child/victim some needed help. Choice D is incorrect. This is a correct statement. Poisonous materials should always be stored away from children and must be locked. Last Updated - 12, Nov 2021

You are the nurse performing education for a patient with AIDS at the community clinic. Which of the following statements is an example of appropriate teaching? A. "Do not wash your dishes with your roommate's dishes." B. "Clean all utensils and dishes before reusing them." C. "Do not use the same shower or toilet as your roommate." D. "Hand sanitizer is not necessary unless you plan on touching someone else."

Explanation Choice B is correct. Stagnant water and food particles can be a breeding ground for pathogenic microorganisms. A patient with an AIDS diagnosis is susceptible to contracting illness/infections more quickly due to the deficiency in his/her immune system. The focus of education should include measures to protect the patient from contracting illnesses from others. Choices A and C are incorrect. Washing dishes with someone else's or sharing bathroom facilities does not protect the patient or the roommate from illness or spread of disease. Choice D is incorrect. Using hand sanitizer is recommended for all people to help prevent the spread of germs. NCSBN Client Need Topic: Physiological Integrity, Subtopic: Reduction of Risk Potential Last Updated - 09, Feb 2022

The nurse is caring for a client scheduled for surgery. The nurse should notify the primary healthcare physician (PHCP) of which laboratory abnormality? See the table below. A. Sodium level B. Potassium level C. BUN D. Creatinine

Explanation Choice B is correct. The client's potassium is considerably low and should be reported to the provider. Choices A, C, and D are incorrect. The client's sodium level is marginally low and does not pose a threat compared to the low potassium. The BUN of 5 mg/dL does not pose a problem to the client as an elevation would be indicative of renal insufficiency. This creatinine level is normal and not an issue. Last Updated - 15, Feb 2022

The nurse is obtaining vital signs for a client who has acquired immune deficiency syndrome (AIDS). Prior to entering the room, the nurse should do which of the following? A. Wear gloves and a gown. B. Perform hand hygiene. C. Review the client's viral load. D. Obtain a disposable stethoscope.

Explanation Choice B is correct. When caring for a client who has AIDS, the nurse should maintain standard precautions. Applying PPE such as a gown, pair of gloves, and mask would be inappropriate. Standard precautions require appropriate hand hygiene and other PPE as needed. Choices A, C, and D are incorrect. The diagnosis of AIDS requires standard precautions which mandate appropriate hand hygiene. It would be inappropriate for gowns or gloves to be worn during client care. Assessing the client's viral load prior to obtaining vital signs would not change the fact that this client requires standard precautions. A disposable stethoscope and blood pressure cuff may be useful for a client with contact precautions, but it would not be necessary for a client with AIDS. Additional Info For a client with standard precautions, hand hygiene is required before and after client care. The nurse may use alcohol-based hand sanitizers only if the hands are not visibly soiled. Another exception to the use of alcohol-based hand sanitizers is if the client has a pathogen such as C. difficile, which requires that the hands be washed with soap and water. Gloves should only be worn when contact with mucous membranes, blood, or non-intact skin will be anticipated. This type of contact is not expected during the collection of vital signs. Last Updated - 15, Jan 2022

Which of the following healthcare providers are responsible for documenting care provided to a patient? A. The LPNs should document the care that they provided and the care that was given by unlicensed assistive staff. B. The registered nurse must document all of the care that is provided by the nursing assistants because they are accountable for all care. C. All staff members should document all of the care that they have provided. D. All staff should document all of the care that they have provided but since the registered nurse is the only independent practitioner, the RN signs it.

Explanation Choice C is correct. All staff members, including unlicensed assistive staff like nursing assistants, document and sign all of the care that they have personally provided. For example, the nursing assistants will document the vital signs that they have taken; the licensed practical nurses will document all of the treatments and medications that they have given to the patient, and the registered nurse will document nursing diagnoses and assessments that they have completed. There is an old saying among healthcare professionals that have been passed on to new generations. The saying is, "I don't care what you did; if you didn't document it, you didn't do it." Documentation is an essential part of patient care. A patient's complete medical record is a legal document. Proper documentation means 1. The person who provided care should document what care/treatment/medication was given and how the patient responded. 2. If care is delegated to another person, it should be noted to whom the responsibility was assigned; proper documentation AND follow-up should be done. Choice A is incorrect. Each person providing care should personally document the attention that he/she provided. Choice B is incorrect. Although the RN or charge nurse is responsible for making sure tasks are delegated to the appropriate personnel, only the person who performs the care should document the care that was provided. Choice D is incorrect. The person providing care should document the care followed by his/her signature. NCSBN Client Need Topic: Safe and Effective Care Environment, Subtopic: Coordinated Care, Roles and Functions of the Nurse Last Updated - 23, Jan 2022

Which hazardous gas can be identified in the home with a simple and relatively inexpensive monitor and alarm similar to a smoke alarm? A. Ozone B. Nitrous oxide C. Carbon monoxide D. Carbon dioxide

Explanation Choice C is correct. Carbon monoxide is a hazardous gas that can be identified in the home with a simple and relatively inexpensive monitor and alarm similar to a smoke alarm. This odorless and colorless gas can be deadly, so it is recommended that all homes have a carbon monoxide alarm. Choice A is incorrect. Although ozone can be present and harmful in buildings, there is not a simple and relatively inexpensive monitor and alarm for ozone that is similar to a smoke alarm. Choice B is incorrect. Although nitrous oxide is a medical gas, there is not a simple and relatively inexpensive monitor and alarm for nitrous oxide that is similar to a smoke alarm. Choice D is incorrect. Although carbon dioxide is a gas, there is not a simple and relatively inexpensive monitor and alarm for carbon dioxide that is similar to a smoke alarm. Last Updated - 10, Feb 2022

When teaching medication safety to the parent of a toddler, which statement by the parent would be a cause for concern? A. "I always check to make sure the safety cap clicks when I close it." B. "We store all of our medicines on a really high shelf." C. "To get her to take her medicine, we tell her it's candy." D. "We store our medicines and vitamins together."

Explanation Choice C is correct. Children should never be told that medication is candy. Choice A is incorrect. It is an excellent practice to make sure safety caps are adequately screwed on; therefore, this option is not a cause for concern. Choice B is incorrect. This answer does not reflect a reason for concern. Choice D is incorrect. There is nothing wrong with storing vitamins and medicines together as long as they are both kept out of the reach of children. NCSBN Client Need Topic: Safe and Effective Care Environment, Subtopic: Safety and Infection Control; Pediatric Health Promotion and Safety Last Updated - 11, Nov 2021

The RN provides discharge instructions to a patient with a diagnosis of paroxysmal atrial fibrillation and a history of macular degeneration. Which instruction would be most important to promote this patient's safety within the home? A. Instruct the caregiver to place items in the patient's central point of vision. B. Avoid eating green, leafy vegetables while on warfarin therapy. C. Keep a large-print list of emergency contact numbers posted on the refrigerator. D. Use an electronic blood pressure cuff to monitor and record blood pressure daily.

Explanation Choice C is correct. Of the options listed, keeping a large print list of emergency numbers would be most important to promote the patient's safety in the home. Macular degeneration is a progressive visual problem that affects central vision and can make it difficult to visualize text. Patients should be encouraged to use large print resources, ensure adequate lighting, and use magnifying tools to maximize their remaining vision. Choice A is incorrect. Macular degeneration affects central vision, not peripheral. Placement of items in the central line of view would be appropriate for a visual problem that affects peripheral vision, such as glaucoma. Choice B is incorrect. Green leafy vegetables can interfere with the coagulation and effectiveness of warfarin. Although the nurse would expect this patient to be on anticoagulant therapy due to atrial fibrillation, the question does not specify warfarin is ordered, so this teaching may not be appropriate for this patient. Additionally, a diet rich in fruits and dark green leafy vegetables is recommended for slowing the progression of macular degeneration. Choice D is incorrect. The question does not mention this patient has any issues with blood pressure. Although it would not be harmful to monitor blood pressure, this would not be the best action to promote the patient's safety at home based on the given information. Last Updated - 10, Feb 2022

The nurse needs to assess the use of complementary and alternative medicine (CAM) because: A. Patients should be warned that most CAM therapies are potentially dangerous B. Additional treatment may not be needed if the patient is using CAM C. CAM therapy could interact with prescription and over-the-counter medications D. Most CAM therapies are essentially ineffective

Explanation Choice C is correct. One key concept to remember when dealing with alternative therapies is that natural does not always mean "better or safe." Some herbal products contain ingredients that may interact with prescription drugs. For example, patients taking medications with potentially dangerous adverse effects, such as insulin, warfarin, or digoxin, should be warned to never take dietary supplements without first discussing their needs with a physician. Complementary and alternative medicine (CAM) is comprised of an incredibly diverse set of therapies and healing systems. CAM is considered to be outside the mainstream of healthcare. From a therapeutic perspective, much of the value of CAM therapies is their ability to reduce the need for medications. Choice A is incorrect. Most CAM therapies are not considered dangerous. However, patients should discuss CAM therapy with their physician before beginning treatment. Choice B is incorrect. While some therapies may reduce the need for conventional medical intervention, the use of CAM does not always reduce the need for that type of response. Choice D is incorrect. CAM therapies are considered valuable in the prevention and treatment of disease. For example, meditation, massage, yoga, and prayer have been used for centuries to treat the body and mind. NCSBN Client Need Topic: Physiological Integrity Subtopic: Pharmacological Therapies Last Updated - 14, Feb 2022

Which of the following actions is most effective at reducing the incidence of health-care-associated infections? A. Screen all newly admitted clients for colonization or infection with MRSA. B. Develop policies that automatically start antibiotic therapy for clients colonized by multi-drug resistant organisms. C. Ensure that dispensers for alcohol-based hand rubs are readily available in all client care areas of the hospital. D. Require nursing staff to wear gowns to change wound dressings for all clients. Submit Answer

Explanation Choice C is correct. Since the hands of healthcare workers are the most common means of transmission for infection from one client to another, the most effective method of preventing the spread of disease is to make supplies for hand hygiene readily available for staff to use. Reducing the risk of healthcare-associated infections is the responsibility of every healthcare worker. Following standard precautions for all patients is the easiest and most effective way of preventing the spread of disease. Choice A is incorrect. Although some hospitals have started screening newly admitted clients for MRSA, there is no evidence that this decreases the spread of infection. Choice B is incorrect. Because the administration of antibiotics to individuals who are colonized by bacteria may promote the development of antibiotic resistance, antibiotic use should be restricted to clients who have clinical manifestations of infection. Choice D is incorrect. Wearing a gown to care for clients who are not on contact precautions is unnecessary. NCSBN Client Need Topic: Safe and Effective Care Management, Subtopic: Safety and Infection Control Last Updated - 27, Oct 2021

You are educating a mother about the association between autism and the MMR vaccine. You know that the mother understands your instructions when she says: A. "My child should not get the vaccine since it is known to cause autism." B. "My child should get the individual immunizations for measles, mumps, and rubella since the individual vaccines do not cause autism." C. "My child should get the MMR immunization since there is no evidence that it causes autism." D. "My child should not get the immunization because it contains mercury." Submit Answer

Explanation Choice C is correct. The CDC and experts at the American Academy of Pediatrics agree that there is no credible evidence that the MMR vaccine causes autism spectrum disorder (ASD). Some of the concerns may be because children typically get the MMR vaccine at about the same time that signs of ASD appear. In fact, in 2013, the CDC conducted a study that showed that vaccines do not cause ASD. This study showed that the antigens in vaccines that produce antibodies were the same between children diagnosed with ASD and children without ASD. Currently, there are no MMR vaccines that contain mercury. Although there is no evidence that mercury causes ASD, mercury was removed from all childhood vaccinations by 2001. The exception to that is that some multi-dose flu vaccines may still contain traces of thimerosal (a chemical containing mercury). Choices A, B, and D are incorrect. These statements are incorrect. NCSBN Client Need Topic: Health Promotion and Maintenance, Sub-Topic: Health Promotion/Disease Prevention, Safety/Infection Control Last Updated - 11, Jan 2022

Select the hazard of immobility and complete bed rest that is accurately paired with one of its preventive measures. A. The accumulation of respiratory secretions: Oxygen supplementation therapy B. Dorsiflexion of the foot: Using a foot board or boots to maintain proper positioning C. Venous stasis: The use of a sequential compression device D. Skin breakdown: The use of a tilt table for clients at risk Submit Answer

Explanation Choice C is correct. Venous stasis, a complication of immobilization and bed rest can be prevented with the use of a sequential compression device (SCD), anti-embolic stockings, client positioning, range of motion exercises, and active leg exercises in bed to promote venous return and to prevent venous stasis, deep vein thrombosis, and pulmonary emboli. Choice A is incorrect. Although the accumulation of respiratory secretions is a hazard of immobility, it is not prevented with oxygen supplementation therapy. It can be prevented with adequate fluid intake, coughing, and deep breathing exercises. Choice B is incorrect. Dorsiflexion of the foot is not a hazard of immobility and bed rest (however, plantarflexion and foot drop are hazards). Plantar flexion can be prevented with a footboard or boots to maintain proper positioning and exercising the feet. Choice D is incorrect. Although skin breakdown and pressure ulcers are hazards of immobility, they are not prevented with a tilt table. Skin breakdown and pressure ulcers are prevented with good skincare, turning and positioning, and other preventive measures such as maintaining proper nutrition and avoiding the forces of pressure, friction, and shearing. Last Updated - 06, Sep 2021

The charge nurse is performing safety rounds on clients in the nursing unit. Which observation requires follow-up? A client with A. an indwelling urinary catheter hanging from the bed frame. B. right-sided weakness with their cane on the left side of the bed. C. a history of falling given a bedside commode. D. a belt restraint was applied and secured over the chest.

Explanation Choice D is correct This observation requires follow-up because a belt restraint should be applied to the client's waist - not the chest. It is inappropriate to have a belt restraint secured over the client's abdomen or chest. Choices A, B, and C are incorrect. These observations do not require follow-up. An indwelling catheter tubing should hang freely from the bed frame. The tubing should be without any kinks or loops. If a client has right-sided weakness, their ambulation device (cane, walker, etc.) should be placed on their stronger side. A client with a history of falling is at an increased risk for future falls and placing a bedside commode is a measure to reduce falls (by decreasing the client's distance to ambulate to the bathroom). Additional Info Belt restraints may be warranted for confused or impulsive clients who are continually trying to get out of bed or a chair after repeated redirection, when it's unsafe for them to get up unassisted. Belt restraints should be applied over clothing and secured over the client's waist. The restraint should be anchored to an immovable part of the bed or the chair. Last Updated - 22, Jun 2022

The occupational health nurse administers a Mantoux intradermal skin test. Which teaching is correct regarding the results of this test? A. A positive result indicates the patient has tuberculosis. B. Redness around the injection site within 24 hours will be recorded as a positive result. C. The patient will return in one week for visualization of the injection site. D. A 3 mm induration after 48 hours indicates a negative result.

Explanation Choice D is correct. 3mm induration after 48 hours would be recorded as a negative result. Induration, less than 5mm after 48-72 hours, is considered an adverse reaction. Choice A is incorrect. A positive Mantoux intradermal skin test result does not necessarily mean the patient has tuberculosis. A positive result indicates the patient has been exposed to TB and only confirms the presence of antibodies. A patient who tests positive for this test would need additional testing to confirm or rule out tuberculosis. Standard tests to establish a positive Mantoux test include a chest x-ray and sputum culture. Choice B is incorrect. Localized redness occurring the day of the injection is healthy and would not indicate a positive reaction. Choice C is incorrect. The patient should return in 48-72 hours to have the site assessed for induration, not one week later. If the patient is unable to return at the appropriate time, the test needs to be repeated. NCSBN Client Need Topic: Infectious disease, Subtopic: Health screening, expected actions/outcomes Additional Info Last Updated - 26, Jan 2022

A nurse is conducting infection control assessments on the nursing unit. Which client is at the greatest risk for infection? A client A. withdrawing from alcohol and is malnourished. B. receiving methylprednisolone for an asthma exacerbation. C. has an external urinary catheter device for urinary incontinence. D. receiving total parenteral nutrition (TPN) via a central line.

Explanation Choice D is correct. A central line is a significant risk factor for a client to develop a central line-associated bloodstream infection (CLABSI). This occurs because of suboptimal sterile technique during insertion and/or inappropriate dressing changes. Additionally, TPN is a risk factor as the high glucose content makes the client more likely to develop a bacterial or fungal infection. TPN increases the risk for a CLABSI compared to solutions such as 0.9% saline. Choices A, B, and C are incorrect. A client withdrawing from alcohol has a risk for delirium tremens but not a risk for an infection. Further, being malnourished increases the risk of impaired skin integrity but not an infection. Methylprednisolone is a corticosteroid that, if used intermediate to long term, may increase the risk for infection. Asthma itself is a chronic disease but not one that raises the risk for infection. An external urinary catheter is preferred over an internal catheter as this substantially decreases the risk for cystitis. This is a non-invasive way to collect urine. Urinary incontinence may raise the risk factor for cystitis, but this depends on the type (stress, urge, overflow). Standard precautions are executed for all clients and consist of appropriate hand hygiene. By adhering to standard precautions, the nurse may reduce the risk of a client getting a healthcare-acquired infection. Other strategies that a nurse may employ to reduce the risk of infection include - - Chlorhexidine baths for those immobile or having a central line. - Minimizing the use of internal urinary catheters. - Surgical asepsis when completing a central line dressing change. - Absolute adherence to appropriate hand hygiene. Last Updated - 02, Sep 2022

You are taking care of a client with moderate to severe dementia. Select the nursing intervention that insures and protects the client's safety in terms of bathing. A. Ensure that there is a scatter rug outside the shower to prevent the collection of water which could lead to a slip and fall. B. Check and ensure that the bathing water for the client is no more than 101 F degrees in order to prevent client burns. C. Clients with dementia should be encouraged to shower in privacy and without supervision so they do not become hostile. D. Never allow the client to remain in the tub alone without monitoring and supervision so that accidents do not occur. Submit Answer

Explanation Choice D is correct. Bathing safety is highly important. Many accidents occur in the bathrooms of healthcare facilities and the homes of clients. You should never allow the client to remain in the tub alone without monitoring and supervision because accidents can and do occur. Other aspects of bathing safety include the presence and use of assistive bathing devices like grab bars and shower chairs, checking and ensuring that the water in the shower is at an appropriate temperature (less than 110 F degrees to prevent burning), and monitoring the client when they are taking a tub bath or shower bath. Choice A is incorrect. You would not ensure that there is a scatter rug outside the shower to prevent the collection of water because scatter rugs are a safety hazard in themselves so they would only increase the risk of slipping and tripping. Choice B is incorrect. You would not check and ensure that the bathing water for the client is no more than 101 F degrees to prevent client burns, but you would ensure that the temperature is no more than 110 F degrees. 101 F degrees is an uncomfortable, cold temperature for bathing water, and water that is too hot can be harmful and can even cause burns. Choice C is incorrect. Clients with dementia should not shower in privacy and without supervision; they must be closely monitored and supervised at all times. Last Updated - 13, Nov 2021

The community health nurse is evaluating an individual's risks of developing Hepatitis A. The nurse knows which age group is at the highest risk of acquiring Hepatitis A? A. Newborns B. The elderly, ages 60 to 90 C. Teenagers older than age 15 D. Children older than 1 year but younger than 15 years

Explanation Choice D is correct. Children who are preschool and elementary school age and those younger than 15-years-old are at the highest risk of developing Hepatitis A virus (HAV). Hepatitis A is usually contracted by ingesting stool containing the virus. Since it's transmitted through the fecal-oral route, children are more prone as they may not practice proper hand hygiene. Often, children contract the infection by eating/drinking food/water (fruits, vegetables, ice, water) contaminated by stools containing HAV, being carried by the infected person that does not wash their hands after using the restroom or traveling to a developing country without being vaccinated for hepatitis A. Outbreaks may occur in child care centers. The CDC recommends Hepatitis A vaccine to the following groups: All children aged 12-23 months Unvaccinated children and adolescents aged 2-18 years International travelers Homeless people Men who have sex with men Those engaged in recreational drug use Those with HIV Those with chronic liver disease Those at risk for exposure during their work. Pregnant women at risk for Hepatitis A Hepatitis A vaccine is administered in a two-dose schedule. It is recommended to all children at one year of age (i.e. 12 to 23 months) and as a catch-up vaccine for all unvaccinated children and adolescents 2 to 18 years. Choice A is incorrect. Newborns are not the group most likely to develop Hepatitis A. Newborns and those less than age 12 months tend to harbor passively acquired maternal antibodies (passively transferred via the placenta). Routine Hepatitis A vaccination is not recommended to children younger than 12 months because their immune systems are not mature. Vaccines before 12 months of age might result in a suboptimal immune response. However, those children at 6-11 months who travel internationally should be given the vaccine. Choice B is incorrect. These elderly age groups do not have more occurrences of Hepatitis A than children under 15-years-old. However, the elderly tend to have more severe symptoms if they do develop Hepatitis A. Choice C is incorrect. Teenagers older than 15 years of age are not at the highest risk of developing Hepatitis A. Preschool to elementary school-aged, and those younger than 15-years-old are at the highest risk. NCSBN client need Topic: Health Promotion and Maintenance, Health Screening Last Updated - 26, Dec 2021

The nurse working with geriatric clients understands that falls are likely to occur in elderly patients who are: A. Living on disability insurance B. In their 80s C. Living in their own home D. Hospitalized

Explanation Choice D is correct. Unfamiliar surroundings is a significant risk factor for falls, especially in the elderly. The hospitalized patient may become confused or bump into furniture, which could result in a fall. Age-related changes may affect the mobility and safety of older adults. For example, decreased muscle strength, reduced balance, and osteoporosis put older adults at risk for falls and fractures. For health promotion, the nurse assesses the musculoskeletal functioning of the older adult and identifies any risk factors that may contribute to falls or the ability of the older adult to perform ADLs. Health promotion interventions often include providing information about the risk factors for osteoporosis and the importance of adequate intake of calcium and vitamin D. Choice A is incorrect. An individual's source of income has no bearing on the risk of falls. Choice B is incorrect. While age-related changes may cause weakness and a slowed reflex response, age is not the most likely risk factor for falls among the available answer choices. Choice C is incorrect. An elderly client living in his own home will be less likely to fall than a client who is in unfamiliar surroundings. NCSBN Client Need Topic: Safe and Effective Care Environment, Subtopic: Safety and Infection Control, Physiological Aging Last Updated - 17, Dec 2021

The nurse is caring for assigned clients. Which client should be evacuated first during a fire? A client with A. below-the-knee amputation receiving patient-controlled analgesia. B. acute respiratory distress syndrome receiving mechanical ventilation. C. advanced dementia receiving enteral feedings and intravenous fluids. D. acute glomerulonephritis with an indwelling urinary catheter.

Explanation Choice D is correct. When evacuating from an internal disaster, the nurse should first evacuate the most ambulatory client. The client with acute glomerulonephritis only has one device, and the nurse can quickly change the system to a leg bag or instruct the client to keep the bag below their bladder. Choices A, B, and C are incorrect. The client with a below-the-knee amputation will require significant resources to mobilize. Further, the client's PCA device must be secured before evacuation. The client receiving mechanical ventilation will require manual ventilation and oxygen. Thus, requiring a significant number of resources. Finally, the client with dementia receiving enteral feedings and IV fluids must have their devices clamped and locked before evacuation. This client also is unlikely to effectively comprehend evacuation instructions and should be supervised. Additional Info For a fire, the nurse should first evacuate the client(s) closest to the fire. Then, the nurse should evacuate the clients that are the most ambulatory. These individuals may assist others with ambulation out of the facility. Last Updated - 11, Dec 2022

The nurse is conducting an in-service for nursing students. It would be appropriate for the nurse to state which of the following procedures requires a sterile technique? Select all that apply. A. Changing the dressing for a central line B. Inserting an indwelling urinary catheter C. Removing a peripheral vascular access device D. Suctioning an endotracheal tube with in-line suction. E. Inserting a nasogastric tube (NGT) Submit Answer

Explanation Choices A and B are correct. Dressing changes of central lines requires a sterile technique. Central lines include ports, peripherally inserted central catheters, and intrajugular access. An indwelling urinary catheter insertion requires a sterile technique to prevent urinary infection. Choices C, D, and E are incorrect. Removing a peripheral vascular access device does not require a sterile technique. The nurse needs to wear clean gloves during the procedure. It is not necessary to use a sterile technique when using in-line suctioning. This is a closed-loop system, so the endotracheal tube should not be contaminated by the nurse touching it. Inserting a nasogastric tube requires a clean technique, not sterile. Additional Info Specific procedures require the nurse to use a sterile technique (also known as surgical asepsis). This technique intends to eliminate all microorganisms from an object or area, including pathogens and spores. Procedures requiring this approach include central line dressing changes, insertion of an indwelling urinary catheter, and suctioning a tracheostomy. Last Updated - 19, Nov 2022

The nurse is discussing infection control with a group of nursing students. It would be correct to state that droplet precautions are used for which condition? Select all that apply. A. Influenza B. Viral meningitis C. Pertussis D. Hepatitis C E. Lyme disease

Explanation Choices A and C are correct. Conditions requiring droplet precautions include influenza and pertussis. Choices B, D, and E are incorrect. Viral meningitis in adults, Hepatitis C, and Lyme disease are not spread by droplets and require only standard precautions. Meningitis may be secondary to bacteria [Neisseria meningitidis (meningococci) or E.coli, or Streptococcus pneumoniae (pneumococci)] or viruses (enteroviruses are the most common cause. Rare viral causes include mosquito-borne viruses, herpes simplex viruses, mumps). Bacterial meningitis with meningococci requires droplet precautions because meningococci spread through large droplets. Clients with meningococcal meningitis should be placed on droplet precautions (private room, mask) until they have completed 24 hours of appropriate antibiotic treatment. Viral meningitis and pneumococcal meningitis do not require droplet isolation. In adults with viral meningitis, standard precautions are sufficient. In infants and young children, viral meningitis requires contact precautions as well. Since most viral meningitis cases are due to enteroviruses that may be passed in the stool, clients with viral meningitis should be instructed to wash their hands thoroughly with soap and water after using the toilet. Additional Info Personal Protective Equipment (PPE) required for a client with droplet precautions is a surgical mask. The risk of transmitting a droplet pathogen is likely when the nurse is within three feet of the individual infected. Conditions requiring droplet precautions include: Rubella Influenza Pertussis Bacterial meningitis Pneumonic plague Diphtheria (Pharyngeal) Mumps Rhinovirus

Why should a patient's family not be present during an interview about violence? Select all that apply. A. The patient may not answer questions related to the family member which may be perceived as insensitive or inappropriate. B. The family member may be the perpetrator of abusive behavior, which may cause the patient to be hesitant to provide honest answers. C. The patient may feel uncomfortable speaking openly with a relative present. especially if that person has contributed to the patient's stress. D. The family member may be embarrassed or ashamed by the patient's actions or statements and try to withhold or change the facts. Submit Answer

Explanation Choices A, B, C, and D are all correct. Individuals who have been in violent situations often decline to answer questions related to a family member, which could be perceived as insensitive or inappropriate for fear of retaliation. This is especially true if the family member is the perpetrator of violence. Patient interviews are done in private, including without significant others or anyone else who may or could be a perpetrator. Healthcare providers should never make assumptions about who may or may not be a perpetrator of violence. Some family members try to withhold or suggest that the patient's recall of events is incorrect to prevent being personally embarrassed by the patient's answers. NCSBN Client Need Topic: Psychosocial Integrity; Subtopic: Violence Victimization and Perpetration Last Updated - 25, Jan 2022

Which of the following statements regarding mass casualty events are true? Select all that apply. A. Mass casualties are events that overwhelm local medical capabilities. B. When a mass casualty occurs, there is a need to increase the staff at the hospital. C. Many local agencies will collaborate to handle a mass casualty situation. D. An example of a mass casualty event is a fight between visitors in the intensive care unit.

Explanation Choices A, B, and C are correct. A is correct. Mass casualties are events that overwhelm local medical capabilities. Another term for mass casualty events is a disaster. B is correct. When a mass casualty occurs, there is a need to increase the hospital staff to provide safe client care to everyone involved in the event. C is correct. Many local agencies will collaborate to handle a mass casualty situation. Collaboration between multiple agencies and health care facilities will be necessary to provide safe care and proper handling of the crisis. Choice D is incorrect. A fight between visitors in the intensive care unit is a security and local law enforcement issue, not a mass casualty event. NCSBN Client Need: Topic: Physiological Integrity, Subtopic: Risk potential reduction; Prioritization, delegation, and leadership Last Updated - 14, Feb 2022

Which of the following statements are true regarding infection with Diptheria? Select all that apply. A. The agent of infection is Corynebacterium diphtheriae. B. The incubation period for this infection is 2 to 5 days. C. Transmission of the cutaneous diphtheria is via direct contact with the infected person. D. The communicable period for diphtheria is 3-6 days. Submit Answer

Explanation Choices A, B, and C are correct. A is correct. The agent of infection is Corynebacterium diphtheriae. B is correct. The incubation period for diphtheria is 2 to 5 days. C is correct. Diphtheria can manifest in cutaneous or pharyngeal forms. Large droplets can spread pharyngeal diphtheria, so one must use droplet precautions. Transmission of cutaneous diphtheria is via direct contact with the skin sores of an infected person, carriers, or contaminated articles. Choice D is incorrect. The communicable period refers to the duration for which the person can remain infectious. The communicable period for diphtheria varies but is usually 2 to 4 weeks. NCSBN Client Need: Topic: Health Promotion and Maintenance, Subtopic: Infectious disease Last Updated - 13, Feb 2022

The emergency department charge nurse was notified of a mass shooting at a nearby shopping mall. The charge nurse should take which action to prepare for the surge in clients? Select all that apply. A. Work to arrange timely discharge and admission for appropriate clients. B. Establish a holding area for discharged clients not able to go home. C. Modify the nurse/client ratio to accommodate the surge levels. D. Instruct staff to switch from electronic to paper documentation. E. Prepare to provide frequent updates to local media.

Explanation Choices A, B, and C are correct. During an external disaster such as a mass shooting, it is reasonable for the nurse to anticipate a surge in clients. To accommodate the surge of individuals, the nurse should advocate for the timely disposition of clients (either admission or discharge) to clear up necessary space. If a client is discharged but cannot leave until the transportation is arranged, they should be placed in a designated discharge area. The charge nurse will need to modify the nurse/client ratio as the influx in clients will require more staffing resources. Choices D and E are incorrect. Switching documentation methods from electronic to paper would jeopardize client safety. This also would decrease efficiency. This would be an inappropriate action. The charge nurse should not be responsible for frequently updating the media. This would be the responsibility of the facilities administration. Additional Info Key recommendations when preparing for a surge in clients Check and test communications systems Open the incident command center and review action plans Arrange for as many safe discharges as possible Modify the nurse/client ratio to one that is safe and practical Clear the emergency department (ED) by having the physician determine the client's disposition (discharge or admit) Consult the medical director regarding elective surgeries and surgical support staff Notify ancillary departments (laboratory and blood bank) Last Updated - 06, Jul 2022

The nurse is performing a fall risk assessment on a group of clients. It would be appropriate for the nurse to identify the client at risk for falls who Select all that apply. A. older than 65 years of age. B. has a history of two previous falls C. taking oral antibiotics. D. experiences postural hypotension. E. experiencing chemotherapy-related nausea. Submit Answer

Explanation Choices A, B, and D are correct. Risk factors for falls include age over 65 years, documented history of falls, postural hypotension, and unfamiliar environment. Falls among older adults are the most common cause of hospital admissions for trauma. Choice C and E are incorrect. Medications such as diuretics, tranquilizers, sedatives, hypnotics, and analgesics are risk factors for falls. Chemotherapy that causes a common side effect of nausea does not increase the fall risk. Neither do oral antibiotics. Additional Info Fall Risk Factors • History of falls • Advanced age (>80 years) • Multiple illnesses • Generalized weakness or decreased mobility • Gait and postural instability • Disorientation or confusion • Use of drugs that can cause increased confusion, mobility limitations, or orthostatic hypotension • Urinary incontinence • Communication impairments • Major visual impairment or visual impairment without correction • Alcohol or other substance use • Location of patient's room away from the nurses' station (in the hospital or nursing home) • Change of shift or mealtime (in the hospital or nursing home) Ignatavicius, D., Workman, M. L. (102020). Medical-Surgical Nursing, 10th Edition. Last Updated - 22, Sep 2022

The nurse is discussing infection control with a group of nursing students. Which conditions require contact precautions? Select all that apply. A. Respiratory syncytial virus B. Psoriasis C. Pediculosis D. Rubella E. Scabies F. Clostridium difficle Submit Answer

Explanation Choices A, C, E, and F are correct. Conditions requiring contact precautions include RSV, pediculosis, Clostridium difficle, and scabies. Pediculosis refers to infestation with head lice. Clostridium difficle is a spore-forming bacteria that causes diarrhea. RSV may be transmitted by the droplet route but is primarily spread by direct contact with infectious respiratory secretions. Droplet precautions are not routinely warranted but are appropriate if the infecting agent is not known. Choices B and D are incorrect. Rubella (German measles) is isolated using droplet precautions. Psoriasis is an autoimmune condition that does not require isolation. Learning Objective Recognize the conditions where contact isolation precautions are used. Understand that hand hygiene can be performed using alcohol-based hand rubs or soap and water; however, in the case of spore-forming bacteria such as Clostridium difficle, hand hygiene must always be performed using soap and water. Additional Info Contact isolation precautions are a type of infection control precautions used to prevent the spread of pathogenic organisms spread through contact. These organisms may spread through contact with infected secretions, fecal matter, large droplets, and bedding. Contact precautions include: Hand hygiene: Hand hygiene must be performed upon entering and exiting the client's room. Alcohol-based disinfectants or cleaning with soap and water are permissible hand hygiene measures. However, only soap and water must be used in the case of Clostridium difficle and Norovirus. Gloves: Put on gloves before entering the room and discard gloves before exiting the room. Gown: Put on a gown before entering the room and discard the gown before exiting the room. Conditions requiring contact isolation precautions Herpes simplex virus (HSV) Varicella-Zoster Virus (VZV) - requires both contact and airborne precautions until all the lesions crust over. Respiratory Syncytial Virus (RSV) - although transmitted by large droplets, the spread is through contact with droplets. Hence, droplet isolation is not necessary for RSV. Enterovirus Scabies Impetigo Abscesses Pediculosis Enteric infections ( Norovirus, Clostridium difficle) Multidrug-resistant bacteria [Methicillin-resistant Staphylococcus aureus (MRSA), Vancomycin-resistant enterococci ( VRE), multidrug-resistant gram-negative bacteria]

The nurse conducts safety rounds within the nursing unit. Which observation requires follow-up? Select all that apply. A. The client's armband was affixed to the bedside table. B. The client's telephone number and name were used as identifiers. C. Multiple blood specimen tubes are labeled before specimen collection. D. A room number is used as an identifier during medication administration. E. Verifies client's name, date of birth, consent, site, and procedure during a time out process. Submit Answer

Explanation Choices A, C, and D are correct. These observations require follow-up because they are unsafe. The client's armband should not be the sole source of client identification but rather collateral information. It should be attached to the client, not a bedside table or bed. Specimen tubes should never be prelabeled. Specimen collection requires appropriate labeling and client verification when obtaining the specimen. A room number cannot be used as a reliable client identifier. Choices B and E are incorrect. These observations reflect appropriate practice and do not require follow-up. A client's telephone number is an acceptable identifier when combined with their name. A time-out procedure should involve the client and participants of the procedure. Elements include the client's name, date of birth, consent, site, and procedure during a time-out process. Additional Info Two reliable client identifiers should be verified for client activities. Acceptable client identifiers include Name Date of Birth An assigned identification number (e.g., medical record number, etc). Telephone number or another person-specific identifier Electronic identification technology coding, such as bar coding or RFID includes two or more person-specific identifiers The nurse should not use identifiers such as the client's room number. Last Updated - 10, Jun 2022

Which of the following is an example of the implementation step of the nursing process? Select all that apply. A. The nurse carefully removes the bandages from a burn victim's arm. B. The nurse assesses a patient to check her nutritional status. C. The nurse forms a nursing diagnosis for a patient with a seizure disorder. D. The nurse repositions a bed-bound patient every two hours to prevent decubitus ulcers. E. The nurse checks the client's insurance coverage at the initial interview. F. The nurse verifies community resources for a patient with dementia.

Explanation Choices A, D, and F are correct. The nursing process involves 5 steps: Assessment, Diagnosis, Planning, Implementation, and Evaluation ( ADPIE). During the implementation step of the nursing process, nursing actions planned in the previous steps ( according to the care plan) are carried out. The purpose of implementation is to assist the patient in achieving valued health outcomes, promote and restore health, and facilitate coping with altered functioning. By implementing the care plan, the nurse aims to assure continuity of care for the patient during hospitalization and in preparation for discharge. Choices B, C, and E are incorrect. The assessment step of the nursing process includes obtaining not only physiological data, but also psychological, sociocultural, spiritual, economic, and lifestyle data as well. The nursing diagnosis step includes not just primary diagnosis but also formulating diagnoses. The North American Nursing Diagnosis Association (NANDA) defines nursing diagnosis as "a clinical judgment about the human response to health conditions/life processes, or a vulnerability for that response, by an individual, family, group or community." The diagnosis is the basis for the nurse's care plan. Often a nursing diagnosis is based on Maslow's Hierarchy of Needs pyramid and helps prioritize treatments. For example, physiological needs (such as food, water, and sleep) are more fundamental to survival than love and belonging, self-esteem, and self-actualization, so they prioritize nursing interventions. Based on nursing diagnoses, the nurse plans the goals ( planning, step 3) to address the patient's diagnosis/ problems through nursing implementation ( step 4). Assessing a patient for nutritional status ( Choice B) or insurance coverage ( Choice E) occurs in the assessment step, and formulating nursing diagnoses ( Choice C) occurs in the diagnostic step. NCSBN Client Need Topic: Safe and Effective Care Environment, Subtopic: Coordinated Care - Implementing the Plan of Care Last Updated - 03, Jan 2022

Which of the following are substantial nursing interventions for a patient who is one-hour post-op from a cardiac catheterization? Select all that apply. A. Administer their regularly scheduled metformin on time. B. Assess the pulse of the extremity distal to the puncture site. C. Position them supine with the head of bed at 45 degrees. D. Monitor for hematoma formation at the puncture site. Submit Answer

Explanation Choices B and D are correct. The nurse must perform a thorough assessment of the perfusion status of the extremity distal to where the puncture was. In a cardiac cath, a sheath is inserted through an artery and snaked up into the heart. This sheath occluded blood flow during the procedure. So, we must monitor the extremity through which they placed the sheath to ensure perfusion returns properly. This includes assessing the pulse, capillary refill, the color of the extremity if there is any pain or numbness, and movement of the extremity. Usually, a femoral artery is used, so we must monitor the perfusion of the foot on the leg that was accessed (Choice B). Monitoring for hematoma formation over the access site is a critical nursing intervention. The most common complication after a cardiac catheterization is bleeding, and the creation of a hematoma shows bleeding under the skin. The nurse should notify the health care provider if she notes a hematoma forming so that they may evaluate the patient. Be sure to monitor for other signs of bleeding as well, especially around the access site (Choice D). Choice A is incorrect. If the patient who is postoperative from a cardiac catheterization has metformin scheduled, the dose should be held for 48 hours post-op. Iodinated contrast used for cardiac catheterization may cause kidney failure. Should such acute kidney failure occur, metformin metabolites can accumulate and cause lactic acidosis. Therefore, metformin should always be held for 48 hours after any procedure that involves iodinated contrast. Choice C is incorrect. Positioning is critical after a cardiac catheterization. For 4 to 6 hours post-op, the head of the bed should be flat or slightly elevated but no more than 30 degrees. Such positioning prevents bleeding and helps the access site from the cardiac catheterization ultimately heal. It is also essential to educate the patient about this so that they will be still and not try to stand up on their own before they are allowed to. NCSBN Client Need: Topic: Health Promotion and Maintenance; Subtopic: Cardiac Last Updated - 10, Feb 2022

The nurse is participating on a committee that is changing the hospital security plan. Which of the following statements by the nurse would be appropriate to make? Select all that apply. A. Open visitation should be implemented in the newborn nursery. B. Visitors should always wear a badge while in the hospital. C. Oral temperatures should be obtained for all visitors. D. Hand sanitizing stations should be offered throughout the facility. E. Disaster drills should be conducted to ensure staff competency. Submit Answer

Explanation Choices B and E are correct. Proper visitor identification is essential in keeping a hospital secure. This allows for rapid identification and a log of all visitors within the facility. Disaster drills should be conducted to ensure that staff is competent with procedures related to certain threats such as mass bioterrorism, active shooter training, or fire. Choices A, C, and D are incorrect. Open visitation should not be implemented in a newborn nursery because of this risk of infant abduction. Visitors may learn the hospital's layout and certain procedures, enabling a more inconspicuous abduction. Checking the oral temperatures of visitors and providing hand sanitizing stations is essential in infection prevention, but they are not pertinent to the hospital's security plan. Additional Info Features of a hospital security plan should include, but are not limited to: ➢ A secure maternity unit with security features such as video surveillance and alarms. ➢ The maternity unit should have access restrictions by approved visitors that are both appropriately identified and logged. ➢ Staff should wear distinguishable badges and uniforms. ➢ Hospital disaster drills should be performed routinely. This helps ensure staff preparedness for fires, bioterrorism, and bomb threats. Last Updated - 29, Nov 2022

Which of the following interventions is a priority for patient safety during care? Select all that apply. A. Proceed with surgeries immediately with no time-out. B. Use two patient identifiers such as name and date of birth. C. Provide documentation, medical terminology, and SBAR for verbal communication. D. Use alarms safely, especially to prevent harm to patients who are at risk for falls. Submit Answer

Explanation Choices B, C, and D are correct. Each of these answer choices reflects safety priorities that should be included in the care of every patient. In the past, hospitals were considered the safest place for sick patients to be. Unfortunately, that assumption is no longer valid as more reports and studies identify the risks, errors, and potential complications that hospitalized patients are exposed to. The Joint Commission has updated the National Patient Safety Goals for hospital care to improve patient safety. To address the risk in health care delivery in hospitals, some of the updated goals include that nurses must: Improve the accuracy of patient identification by using at least two patient identifiers when providing care. Improve the effectiveness of communication among caregivers by using written documentation, approved medical terminology, and SBAR (situation, background, assessment, recommendation) for verbal communication. Improve the safety of using medications by labeling all drugs and adopting practices to reduce the likelihood of patient harm associated with the use of anticoagulation therapy and reduce the adverse patient outcomes associated with medication discrepancies. Reduce the risk of harm associated with clinical alarm systems by using alarms safely, mainly to prevent the risk of falls. Reduce the risk of healthcare-associated infections by implementing CDC or WHO goals to improve hand cleaning, prevention of diseases from central lines, post-surgical infections, and identify clients at risk for developing hospital-acquired infections. Choice A is incorrect. All surgeries must have a "time-out" period to avoid wrong-site surgeries and other complications. NCSBN Client Need Topic: Safe and Effective Care Environment, Subtopic: Safety and Infection Control; Safety Interventions for Hospitalized Patients Last Updated - 23, Dec 2021

The nurse is caring for several clients in a long-term care facility. Which interventions should the nurse implement to reduce the risk of injury from falls? Select all that apply. A. Avoid administering ibuprofen at night B. Secure the call button to the side of the bed C. Keep the bed in the lowest position D. Place fall risk bands on clients at risk of falling E. Reposition clients off of bony prominences every two hours

Explanation Choices B, C, and D are correct. Falls and injuries can be reduced by ensuring the call button are accessible and within easy reach for the client. The call light can be clipped or secured when the client is in bed to prevent falling out of reach. Falls also may be reduced by placing fall risk bands on clients who are at risk. This will alert staff to the client's risk of falling. Setting the bed to the lowest position would reduce the risk of injury in the event of a fall because such positioning keeps the client at a closer distance to the floor. Choices A and E are incorrect. Evening administration of NSAIDs does not increase the risk of falls. On the other hand, the nurse should avoid administering diuretics and laxatives before the client's sleep to reduce urgent bathroom needs. Having to push for urgent bathroom needs may lead to falls. The nurse should complete a fall risk assessment upon admission or within 2 hours of admission to promptly implement fall-prevention interventions for high-risk clients. Waiting 24 hours to complete a fall-risk assessment is inappropriate because a preventable fall may have already happened. Repositioning clients off bony prominences will help prevent pressure ulcers, not falls. Additional Info Universal fall precautions involve • Monitor the client's activities and behavior as often as possible, preferably every 30 to 60 minutes. • Teach the client and family about the fall prevention program to become safety partners. • Remind the client to call for help before getting out of bed or a chair. • Help the client get out of bed or a chair if needed; lock all equipment such as beds and wheelchairs before transferring client's. • Teach clients to use the grab bars when walking in the hall without assistive devices or when using the bathroom. • Provide or remind the client to use a walker or cane for ambulating if needed; teach him or her how to use these devices. • Remind the client to wear eyeglasses or hearing aid if needed. Last Updated - 11, Nov 2022

A client has been diagnosed with disseminated herpes zoster. Which personal protective equipment (PPE) will you need to put on when preparing to assess the client? Select all that apply. A. Goggles B. Gown C. Gloves D. Shoe covers E. N95 respirator F. Surgical face mask Submit Answer

Explanation Choices B, C, and E are correct. Since herpes zoster is spread through airborne means and by direct contact with the lesions, contact and airborne precautions should be followed in case of "disseminated" herpes zoster. This means the nurse should wear an N95 respirator or high-efficiency particulate air filter respirator, a gown, and gloves. Herpes zoster, also known as shingles, is caused by the reactivation of the varicella-zoster virus (VZV), the same virus that causes varicella (chickenpox). Primary infection with VZV causes varicella. Once the illness resolves, the virus remains latent in the dorsal root ganglia. VZV can be reactive later in a person's life and create a painful, maculopapular rash called herpes zoster. Active herpes zoster lesions are infectious, through direct contact with vesicular fluid, until they dry and crust over. People with active herpes zoster lesions should cover their injuries and avoid contact with susceptible people in their household and occupational settings until their wounds are dry and crusted. Choices A and D are incorrect. Goggles and shoe covers are not needed for airborne or contact precautions. Choice F is incorrect. A surgical face mask filters only large particles and will not protect against herpes zoster. NCSBN Client Need Topic: Safe and Effective Care Environment, Subtopic: Safety and Infection Control

The nurse is triaging a child with suspected impetigo. Which action should the nurse take? Select all that apply. A. Initiate droplet precautions B. Set up a decontamination room C. Use a disposable blood pressure cuff D. Initiate contact precautions E. Apply sterile gloves while examining the client Submit Answer

Explanation Choices C and D are correct. Impetigo is a contagious infection of the skin commonly seen in young children. This condition is highly infectious, and the nurse should utilize standard and contact precautions. Part of this involves using disposable client care equipment (blood pressure cuff, thermometer, etc.). Contact precautions require the nurse to wear a gown and gloves when engaging in client care. Choices A, B, and E are incorrect. Droplet precautions are not appropriate for impetigo. It is spread through contact with the skin, not respiratory droplets. Although very contagious, a decontamination room is not indicated for impetigo. Sterile gloves are not necessary to don during client care. Using this would be a waste of organizational resources. Additional Info Impetigo is a contagious skin condition that is caused by Staphylococcus aureus. This condition is commonly found in young children and typically presents around the face, mouth, and then on the hands, neck, and extremities. The lesions have drainage and then begin to crust. ➢ Medical treatment is antibacterial ointments that should be applied via a sterile cotton tip applicator. ➢ Nursing care focuses on educating the client on hand hygiene, pain control with warm compresses to the affected area, preventing transmission by not sharing linens, etc. Last Updated - 03, Nov 2022

The nurse is observing infection control practices on the nursing unit. Which of the following findings requires follow-up? Select all that apply. A. A client with H. pylori placed on standard precautions. B. Disposable blood pressure cuff used on a client with rotavirus. C. A client's room door kept closed who has rubella. D. A client with influenza ambulating in the hall with a surgical mask. E. Contact precautions for a client with Legionnaires' disease.

Explanation Choices C and E are correct. A client with rubella should be placed on droplet precautions. Droplet precautions do not require that the door be kept closed. The PPE required for rubella includes a surgical mask. Legionnaires' disease is not transmitted person-to-person but rather through infected water or soil. This bacterium requires standard precautions. Choices A, B, and D are incorrect. These observations are correct and do not require follow-up. H. pylori infections are not transmitted from person to person, and thus, standard precautions are appropriate. A disposable blood pressure cuff is appropriate for a client with rotavirus as infected surfaces may transmit this pathogen. It is appropriate for a patient with influenza to ambulate with a surgical mask as this may be spread through infected droplets. Additional Info The minimum personal protective equipment (PPE) required for contact is gloves and a surgical mask. For droplet precautions, it is a surgical mask. Finally, airborne precautions require the client's room door to be kept closed, with negative pressure, and an N95 respirator to be worn.

The nurse is educating staff on infection control. Which of the following statements by the nurse would indicate a correct understanding of infection control guidelines for influenza? Select all that apply. A. Limiting visitation to 30 minutes per day. B. Keeping the door to the client's room closed. C. Wearing a surgical mask when providing care. D. Placing the client in a room at the end of the hall. E. Cleaning common surfaces with 70% isopropyl alcohol. Submit Answer

Explanation Choices C and E are correct. Infected droplets primarily spread influenza. Wearing a surgical mask when providing care is essential. Finally, cleaning common surfaces with a cleaning agent of at least 70% isopropyl alcohol is important as the influenza virus may survive on these surfaces. Choices A, B, and D are incorrect. Visitor restrictions of up to 30 minutes per day would be used for a client receiving brachytherapy. Keeping the client's door closed is reserved for the client on airborne precautions. Placing the client at the end of the hall is inappropriate for an individual with influenza because it would make no difference in infection control. The door should be closed for airborne precautions (to maintain negative pressure) - not droplet precautions. Placing the client at the Additional Info Influenza is a highly contagious respiratory infection. Droplet precautions are implemented for suspected/confirmed cases. The door can remain open as negative pressure is not required. Infection control measures should include frequent hand hygiene, including alcohol-based hand sanitizers, before and after client care. If the hands become visibly soiled, they should be washed with soap and water. A surgical mask is necessary to prevent transmission of the infected droplets. The nurse should wear a surgical mask if care is provided within three feet of the client. If the client should leave the room, they will wear the surgical mask.

The nurse is observing infection control practices in the nursing unit. Which of the following findings require follow-up? A client with Select all that apply. A. H. pylori placed on standard precautions. B. rotavirus provided a disposable blood pressure cuff. C. rubella and their door is kept closed. D. influenza ambulating in the hall with a surgical mask. E. Legionnaires' disease placed on contact precautions.

Explanation Choices C and E are correct. These observations are inappropriate and require follow-up. The door should be closed in airborne isolation precautions, not droplet precautions. A client with rubella should be placed on droplet precautions. The minimum PPE required for droplet precautions is a surgical mask. Legionnaires' disease is not transmitted person-to-person but rather through infected water or soil. This bacterium requires standard precautions. Choices A, B, and D are incorrect. H. pylori infections are transmitted predominantly by contaminated water. H. pylori can spread within the families through close contact among family members. While an intrafamilial transmission is possible through contact with infected family members, there is no data to support using special isolation precautions while caring for a client diagnosed with H.Pylori. Standard precautions are used for H. pylori infections. Rotavirus requires contact precautions. Sanitization of the surfaces is necessary to prevent transmission from contact with the contaminated surfaces. A disposable blood pressure cuff is appropriate for a client with rotavirus, as infected surfaces may transmit this pathogen. Influenza requires droplet precautions. It is appropriate for a client with influenza to ambulate with a surgical mask as it may spread through infected droplets. Learning Objective Recognize the CDC-recommended transmission-based precautions used in infection prevention.

The nurse is inserting a peripheral intravenous catheter. Place each action in the correct order. Apply a tourniquet and palpate a vein for insertion. Clean the skin with approved solution. Tape and secure the IV site. Stabilize the vein below the insertion site (digital traction). Puncture the skin and vein with the stylet. Apply pressure above the insertion site and connect the IV tubing. Observe for blood return and advance the catheter. Submit Answer

Explanation Inserting a peripheral IV requires the application of a tourniquet proximal to the targeted site. The skin should then be cleaned with an approved solution (circular motion), and then digital stabilization should be applied distally. Once the vein is selected and stabilized, the nurse should insert the needle at an appropriate angle. Once blood return is seen in the chamber, the nurse should advance the needle a quarter of an inch and then thread in the catheter. Pressure should be held proximally and the IV site should be flushed to verify patency. The nurse should then secure it in place. Last Updated - 26, Oct 2021 Apply a tourniquet and palpate a vein for insertion. Clean the skin with approved solution. Stabilize the vein below the insertion site (digital traction). Puncture the skin and vein with the stylet. Observe for blood return and advance the catheter. Apply pressure above the insertion site and connect the IV tubing. Tape and secure the IV site.

Place the following elements of evaluation in their correct sequence: Interpreting and summarizing findings Collecting data to determine whether evaluative criteria and standards are met Documenting one's judgment Terminating, continuing, or modifying the plan of care Identifying evaluative criteria and standards Terminating, continuing, or modifying the plan of care Collecting data to determine whether evaluative criteria and standards are met Documenting one's judgment Interpreting and summarizing findings Identifying evaluative criteria and standards Submit Answer

Explanation The correct ordered sequence is: 5, 2, 1, 3, 4. The five classic elements of evaluation are: Identifying evaluative criteria and standards (what you are looking for when you evaluate) Collecting data to determine whether these criteria and standards are met Interpreting and summarizing findings Documenting your judgment Terminating, continuing, or modifying the plan In the 5th step of the nursing process, evaluating, the nurse measures how well the patient has achieved the outcomes specified in the plan of care. When evaluating patient outcome achievement, the nurse identifies factors that contribute to the patient's ability to achieve expected results and, when necessary, modifies the plan of care. The purpose of the evaluation is to allow the patient's achievement of expected outcomes to direct future nurse-patient interactions. NCSBN Client Need Topic: Safe and Effective Care Environment, Subtopic: Coordinated Care - Components of Evaluation Last Updated - 14, Apr 2020


Ensembles d'études connexes

First Year Innovation Exp Midterm Connect 2022

View Set

English Composition - Reading Quiz

View Set

Biology: QUIZ 2: BODY FRAMEWORK AND REPRODUCTION

View Set

FINAL EXAM Business 101 Chapter 9

View Set

Public Speaking Chapter 4: Listening Skills

View Set

IBM Planning Analytics Workspace V2.0.0 Explore Data Home 17/30 testing

View Set

CISSP Official ISC2 practice tests - Domain 2

View Set